Сохранен 558
https://2ch.hk/spc/res/259700.html
Домены arhivach.top и arhivach.site временно не функционируют! Используйте домен ARHIVACH.XYZ.
24 декабря Архивач восстановлен после серьёзной аварии. К сожалению, значительная часть сохранённых изображений и видео была потеряна. Подробности случившегося. Мы призываем всех неравнодушных помочь нам с восстановлением утраченного контента!

Тред тупых вопросов #43

 Аноним 08/02/16 Пнд 21:33:19 #1 №259700 
14549563994250.jpg
Тред вопросов о жизни, Вселенной и всем таком.

Спрашиваем то, за что в других местах выдают путёвку в биореактор. Здесь анонимные ученые мирового уровня критически рассмотрят любые гениальные идеи и нарисованные в Paint схемы.

Прошлый тред
https://2ch.hk/spc/res/254581.html
Аноним 08/02/16 Пнд 21:51:28 #2 №259706 
14549574888160.png
На правах переката перепощу свой вопрос на который внятно никто не ответил.

Есть один конденсатор.
Его поместили в вакуум и подали на пластины напряжение равное или превышающее напряжения пробоя вакуума. Потом между пластинами поместили заземленный диэлектрик диэлектрик разделен на пластины, каждая замкнута на нагрузку, как на пикче. Диэлектрик не касается пластин и перетекание электронов между пластинами через него невозможно.
Вопрос, что будет если пробить насквозь пластины конденсатора вместе с диэлектриком между ними? Тобишь, чтобы в конденсаторе и диэлектрике на одной перпендикулярной прямой была дырка. Пробъет вакуум по этой дырке или нет? Напряженность поля внутри дырки диэлектрика будет равно напряжению пробоя или нет? Можно ли с помощью такой конструкции сделать компактный ускоритель, ну на пол метра длинной, на котором будет работать ПЫЩЬПЫЩЬ ЛУЧЕ-ГАММА-МЕТЕ?

Сразу скажу зачем это надо, казалось бы никаким боком не космически-межгалактический вопрос.
Во первых, йоба ионный двигатель с охуительной тягой. Ведь чем сильнее поле, тем большую энергию приобретают ионы, и тем больший импульс они могут дать. Пока все ограничено напряжением пробоя вакуума.
Во вторых, это adsподкритический управляемый ускорителем реактор для всяких розет и пионэров, чтобы на них можно было ставить более йобистые приборы, требующие большего питания.
Ну и в третьих, ЛАЗОР для ионизации и анализа того что наионизировал.
sageАноним 08/02/16 Пнд 22:51:30 #3 №259722 
>>259706
картинка говно, перересуй
также как и вопрос, поэтому тебе в прошлый раз никто и не ответил.
если ты запихнешь диэлектрик, касается он пластин или нет, напряжение пробоя вырастет.
нахуй короче, дальше читал, но нахуй.
сам жрешь говно и других пытаешь им накормить.
Аноним 09/02/16 Втр 09:55:14 #4 №259808 
Что происходит на границе вселенной? Там что прост пространство появляется?
Аноним 09/02/16 Втр 10:04:25 #5 №259809 
>>259808
>Что происходит на границе вселенной?
Там батя на карачках сидит, и в щель заглядывает.
Аноним 09/02/16 Втр 10:16:54 #6 №259810 
>>259809
Пруфы?
Аноним 09/02/16 Втр 11:47:59 #7 №259817 
>>259808
Мы не знаем есть ли граница,т.к. свет туда не дошел.
Аноним 09/02/16 Втр 11:48:35 #8 №259818 
>>259809
Ты ебанутый?Худа ты прилетел?
Аноним 09/02/16 Втр 15:20:27 #9 №259882 
14550204271930.jpg
Псосны, в детстве мучал вопрос, в рефлекторах вторичные зеркала как то ухудшают изображение? Пикрелейтед.
Аноним 09/02/16 Втр 16:19:29 #10 №259887 
>>259882
Да, режут апертуру немного, это очевидно. Изображение не ухудшают, просто площадь собирающей свет поверхности меньше.
Аноним 09/02/16 Втр 16:54:12 #11 №259890 
14550260528820.jpg
>>259887
Аноним 09/02/16 Втр 16:54:59 #12 №259891 
>>259722
Напряжение вырастет, ЭТОЖЕИТАКОЧЕВИДНО.
Но пробъет ли вакуум через дырку или нет я так и не понял.
Аноним 09/02/16 Втр 16:57:54 #13 №259892 
Мне нужно чтобы на расстоянии в один метр некое тело ускорилось до скорости 8км/сек. Как посчитать необходимое ускорение?
Аноним 09/02/16 Втр 17:13:25 #14 №259895 
>>259892
Ты охуел что ли? Равноускоренное прямолинейное движение в 9-м классе проходят. Вместе со всеми ебучими формулами. а=v^2/2S
Аноним 09/02/16 Втр 18:27:26 #15 №259900 
>>259895
>Ты охуел что ли?
>в 9-м классе проходят.

Ты полагаешь, что нормальный человек, окончивший школу лет 15 назад, будет эту хуйню помнить и в голове держать? Кроме тех, случаев, когда он по работе или ученой деятельности с этим связан.

У меня для тебя плохие новости.
Аноним 09/02/16 Втр 18:43:02 #16 №259901 
>>259900
Нормальный человек, окончивший школу лет 15 назад, смог бы сам найти нужные формулы для такой хуйни. Ой, забыл, что у нас норма это большинство.
Аноним 09/02/16 Втр 18:56:38 #17 №259902 
>>259900
>Ты полагаешь, что нормальный человек, окончивший школу лет 15 назад, будет эту хуйню помнить и в голове держать?
Хм, в тот ли ты раздел зашел? Да, нормальный образованный человек вполне будет помнить формулу и смысл прямолинейного равноускоренного движения через 15 лет. Даже моя соседка Аллочка-дурочка помнит, я гарантирую это.
Аноним 09/02/16 Втр 19:04:40 #18 №259903 
>>259895
Получается что в моем случае ускорение должно быть 32000000 метров в секунду. Правильно же?!
Аноним 09/02/16 Втр 19:12:47 #19 №259904 
>>259903
> м/с
Не правильно ты Дядя Федор ускорение считаешь.
Аноним 09/02/16 Втр 19:14:16 #20 №259905 
>>259904
Ты все понял что там секунда квадратная.

Формула точно правильная? А то я посчитал какое ускорение у пули в калашникове и получается охуительные 650000 метров в квадратной секунде.
Аноним 09/02/16 Втр 19:27:28 #21 №259912 
>>259901
>смог бы сам найти нужные формулы для такой хуйни.

С этим соглашусь.

А этот в манямирке >>259902
Плотнячком.
Аноним 09/02/16 Втр 19:29:45 #22 №259913 
>>259912
> в манямирке
Зато там уеб нету.
Аноним 09/02/16 Втр 19:32:29 #23 №259914 
>>259905
Вобщет 405 км/с2 если скорость на выходе 900 м/с
Аноним 09/02/16 Втр 19:41:06 #24 №259920 
14550360662020.png
>>259914
Аноним 09/02/16 Втр 20:12:54 #25 №259952 
>>259912
Эта Аллочка выдрачивает свою аллочку-младшую по физике, есличе.
Аноним 09/02/16 Втр 20:25:07 #26 №259965 
>>259900
Полагаю, что нормальный человек загуглит это за 2 секунды, вместо того, что бы писать в ТТВ и ждать, когда ему ответят. Понимаю, если бы он спросил какую-то хуйню по астрономии, которую сейчас в школе не преподают, еще немного и тут начнут вопросы по таблице умножения задавать.
АНОН СКОЛЬКО БУДЕТ 3х9 ???!!!
Аноним 09/02/16 Втр 20:45:11 #27 №259978 
>>259965
>АНОН СКОЛЬКО БУДЕТ 3х9 ???!!!
42
Аноним 09/02/16 Втр 20:57:04 #28 №259984 
>>259965
27
Аноним 09/02/16 Втр 21:16:00 #29 №259991 
>>259984
Действительно 27. А я сразу подумал 29
Аноним 09/02/16 Втр 22:48:13 #30 №260029 
>>259913
и тянки не дают )))

Аноним 09/02/16 Втр 22:54:31 #31 №260031 
>>259991
вот, о чем и речь
так и себя проверил и другим жирок подрастряс
те я угадал да, или всетаки 29
Аноним 09/02/16 Втр 22:58:04 #32 №260032 
14550478849420.png
>>260031
Гугл говорит что ты прав
Аноним 09/02/16 Втр 23:00:56 #33 №260033 
>>259808
Пока на этот вопрос нет ответа. Нет данных о наличии какой либо границы у вселенной, а также что находится ЗА этой границей.
Аноним 09/02/16 Втр 23:04:49 #34 №260035 
>>260032
Подтверждаю. Сейчас позвонил Перельману, попросил проверить решение. Вначале отказал, но через пару минут перезвонил и сказал что решено верно.

Мимо к. Фмн
Аноним 09/02/16 Втр 23:30:34 #35 №260045 
>>260035
проиграл, наповал.
Сенсация, ученые мирового уровня решили важнешйую проблему арифметики, и изобрели гуглокалькулятор. Сенсация, сенсация, только в двач-таймс, 10 центов, господин купите, всего 10центов
Аноним 10/02/16 Срд 00:31:33 #36 №260063 
>>259900
Ты просто идиот, ведь по-моему диагноз, когда человек забывает элементарные вещи, именно так звучит.
Аноним 10/02/16 Срд 00:32:12 #37 №260065 
>>259808
С какого хуя ты решил, что у вселенной есть граница?
Аноним 10/02/16 Срд 00:55:37 #38 №260069 
>>260065
ТССС. Сейчас снова придет шизик и будет рассказывать, что Вселенная - это сфера.
Аноним 10/02/16 Срд 00:57:23 #39 №260071 
>>260029
Их там тоже нет. :(
Аноним 10/02/16 Срд 00:59:29 #40 №260073 
>>260069
мультивселенная?
Аноним 10/02/16 Срд 01:03:37 #41 №260074 
>>260069
Нет, она на ведро похожа
Аноним 10/02/16 Срд 01:05:18 #42 №260075 
>>260073
Мультивселенную не плохо было бы подвязать на какую-нибудь в принципе проверяемую теорию, а не выковыривать из носа, как козявку.
Аноним 10/02/16 Срд 01:07:03 #43 №260076 
>>260075
Ну извини, скуптик.
Аноним 10/02/16 Срд 02:49:19 #44 №260101 
>>259700 (OP)
Анончик, знаешь ли ты более мощный объект во вселенной чем квазар и если да, поясни за него.
Аноним 10/02/16 Срд 03:30:08 #45 №260106 
>>260101
>более мощный объект во вселенной чем квазар
Очередной научпоп по телику показали?
По сути, если говорить о мощности оптического излучения, то эти ебучие фонарики светят действительно очень ярко. Их видно за 10-13 миллиардов световых лет. С другой стороны есть некие объекты, которые выбрасывают гамма-всплески большей мощности, но в гамма диапазоне, но до конца непонятно, что их рождает. Так что да, в целом можно сказать, что квазары - это самые мощные объекты.
Аноним 10/02/16 Срд 04:19:10 #46 №260111 
>>259887
то есть черной тени посередине не будет?
Аноним 10/02/16 Срд 04:25:44 #47 №260113 
>>259808
Границы неизвестны, есть горизонт из за которого свет не доходит по физическим причинам, но за горизонтом могут быть точно такие галактики как у нас в пределах горизонта.
Аноним 10/02/16 Срд 04:39:49 #48 №260114 DELETED
>>260111
Нет конечно.

Это только на фоточках сделанных с такими абиктивами бывают специфические бокешечки.
Но с телескоп обычно фоткают только на бесконечность и избегают попадания ближних источников света в кадр.

Однако, если сделать фоточку не в фокусной плоскости, то увидишь расплывчатое кольцо.
Аноним 10/02/16 Срд 05:14:00 #49 №260120 
Господа. Тупой вопрос от малообразованного человека. Можете послать нахуй, но хотелось бы все-таки понять. Ученые дядьки говорят, мол, Земля вращается вокруг Солнца (да, я знаю, что вокруг баррицентра, но сейчас это не суть), Солнечная система в рукаве Ориона обращается вокруг центра галактики Млечный путь. Тут все понятно. Далее галактики соединяются в скопления, скопления в сверхскопления а те в свою очередь образуют некие структуры, уже запамятовал как именно называемые. И вот тут я никак не могу понять, почему ученые дядьки говорят о строении вселенной на уровне галактик, их скоплений и так далее, как о единовременном факте. Ведь скорость света предельна, во-всяком случае наши способности наблюдений упираются в нее, и мы видим по сути прошлое. Мало того, блядь, мы видим разные моменты этого самого прошлого! Так как же можно говорить о расположении относительно друг-друга отдельных галактик, а тем более их скоплений, если там разница в сотни тысяч, а то и миллионы световых лет? Получается, что, условная галактика А находилась на этом месте 10 000 световых лет назад, а условная галактика Б, находилась относительно ее на определенном месте, но уже 100 000 лет назад. Это какая-то мозаика из фоток разных эпох! А какова реальная картина мира В ДАННЫЙ МОМЕНТ? Да и вообще, а можно ли говорить о возможности существования этой "реальной" картины?
Размеры Метагалактики, если верить википедии примерно 46 миллиардов световых лет. Т.е. самые дальние объекты, это не просто далеко. Это еще и безумно давно. А дальше, вроде бы должно быть что-то, просто мы не видим ПОКА. А еще дальше сингулярность и начало/конец мира. Это я как-то худо-бедно представляю. Но как можно судить о строении наблюдаемой вселенной, которая для нас является снимками РАЗНЫХ временных участков?
Мне трудно точно сформулировать суть вопроса, надеюсь более-менее понятно излагаю - делаю это, как умею, уж не серчайте. Попробую суммировать: Является ли описываемая модель (принятая на сегодняшний день) вселенной справедливой для каждой отдельно взятой галактики, либо она описывает то, что мы наблюдаем, не обращая внимание на разницу во времени и реально в данный момент она не будет справедлива при переносе наблюдателя куда-нибудь в далекие/давние ебеня?
Или я вообще несу какую-то херню и мне нужно учить матчасть?
Аноним 10/02/16 Срд 05:24:38 #50 №260121 
>>260120
>А какова реальная картина мира
Реальная? Что ты под этим подразумеваешь? Т.к. пространство неотделимо от времени, а мы в этом пространстве-времени находимся, то для нас реально то, что мы видим.
Аноним 10/02/16 Срд 05:39:26 #51 №260122 
>>260121
Ну, смотри. Галактика, даже самая удаленная от нас - это же физический, реальный объект. Она есть, или хотя бы была. Мы говорим: "Есть скопление номер такое-то, куда входят галактики такие-то". Только одна из них миллион световых лет назад, другая пять миллионов и т.д. Ну, условно, разумеется. Относительно чего тогда мы говорим о скоплении? О том, что разные галактики, относительно нас в разное время находились в близкой области вселенной? Какое же это тогда скопление-то?
Для нас, ты говоришь, реально... Для нас реально то, что достижимо, как мне кажется. Марс реален, потому что мы точно знаем, где он находится относительно нас прямо сейчас. То есть, если у меня будет машинка перемещения и я, ткну пальцем в Марс на небе, и скажу "Мне туда", то я туда и попаду. Но если я ткну в Большое Магеланово Облако, то я туда не попаду, потому что его в этом направлении сейчас уже нет. Оно там было черт знает когда. Но для самого себя-то, это Облако есть где-то (при условии, что оно еще не исчезло). Для нас оно в одном месте Вселенной, а для себя в другом. А для наблюдателя в Туманности Андромеды в третьем. И все это еще и во-времени, а не только в пространстве. Так как тут можно судить о строении вселенной?
Аноним 10/02/16 Срд 05:42:40 #52 №260123 
>>260122
фикс*
Или тут нужно четырехмерное мышление и я вообще неправильно рассуждаю и ставлю вопрос?
Аноним 10/02/16 Срд 05:46:20 #53 №260124 
>>260120
>она описывает то, что мы наблюдаем, не обращая внимание на разницу во времени и реально в данный момент она не будет справедлива при переносе наблюдателя куда-нибудь в далекие/давние ебеня?
Это, в какойто степени. То о чем ты говоришь касается в большей степени пространственной модели, звезды не перестанут быть звездами, галактики галактиками.
Более того расстояния достаточно условны, определяемые по косвенным признакам, и определенным допущениям.
Но в принципе это какраз удобно, эдакая машина времени в прошлое.
Аноним 10/02/16 Срд 05:53:57 #54 №260126 
>>260124
Ага, значит все-таки в большей степени условность. Ну, в принципе оно и понятно, мы же не можем наблюдать вселенную извне...
Аноним 10/02/16 Срд 05:54:49 #55 №260127 
>>260122
>Так как тут можно судить о строении вселенной?
Мы видим кусочки одной системы, в разных состояниях в пространстве и времени.
Те если бы к примеру на выскоскоростную камеру засняли взрыв, но при просмотре показывали бы только один сегмент потом другой потом третий (те не весь кадр сразу один за другим, а кусочки кадра)
при определенной фантазии и разумных допущениях можно предположить как примерно что выглядит.

В каком состоянии вселенная сейчас, пространственном, нам точно не известно
Аноним 10/02/16 Срд 06:05:08 #56 №260128 
>>260127
Хм. Да, наверное я просто слишком много требую от приблизительной теории, вы правы.
Аноним 10/02/16 Срд 06:44:58 #57 №260132 
Сможет ли разум спастись, пережить, предотвратить смерть вселенной?
Аноним 10/02/16 Срд 06:46:17 #58 №260134 
>>260132
На всё воля Аллаха.
Аноним 10/02/16 Срд 06:58:02 #59 №260135 
14550766823940.jpg
14550766824071.jpg
>>260111
Если зеркало присобачено на растяжках, то звезды имеют лучики по числу растяжек.
Аноним 10/02/16 Срд 07:11:25 #60 №260136 
14550774859650.webm
14550774859941.webm
>>260120
В общем есть такое интересное. Называется "Космологический принцип", который вещает нам о том, что где бы ты не сидел во Вселенной, ты видишь одну и туже картину, вне зависимости от того, где сидишь и в какое время. Условно говоря там тоже самое, что и здесь. Это что касается общего, что можно погуглить и почитать на досуге. Что касается скорости убегания и собственного движения. Если мы возьмем филаменты, или как их еще называют галактическими нитями, те самые крупные структуры из галактик и скоплений галактик, название которых ты забыл, то эти сопли, развешенные по всей Вселенной настолько огромны, что о о съебе на хоть какое-либо значительное расстояние говорить не серьезно. Где-то конечно болтаются галактики, котроые уже слились в скопления, а мы их видим отдельно, но это очень далекие объекты. И последнее. Есть моделирования. Все они показывают примерно одно и тоже по всей Вселенной.
В целом нет четкой одновременности моментального снимка, если у нас скорость света ограничена. Если мы видим объект в миллионе световых лет назад, то мы его видим таким, каким он был миллион лет назад, сейчас он может быть другой.
Аноним 10/02/16 Срд 07:33:46 #61 №260140 
>>259920
Ну ты 7,62-старикана-то не привлекай.
Аноним 10/02/16 Срд 10:23:02 #62 №260165 
Как могла бы поменялись физические принципы если бы скорость света была бесконечна?
Аноним 10/02/16 Срд 10:29:03 #63 №260166 
>>260165
E=m*c^2
Начнём с того, что материи бы не было.
Аноним 10/02/16 Срд 10:30:25 #64 №260168 
>>260166
А что было бы?
Аноним 10/02/16 Срд 10:30:45 #65 №260169 
>>260168
Вакум
Аноним 10/02/16 Срд 10:34:23 #66 №260172 
>>260169
> Вакум
Это кто?
Аноним 10/02/16 Срд 11:12:24 #67 №260179 
>>260165
>Как могла бы поменялись физические принципы если бы скорость света была бесконечна?
Не выполнялся бы принцип причинности, и еще сто тысяч других принципов. Современная физика стоит на позициях близкодействия:
https://ru.wikipedia.org/wiki/Дальнодействие_и_короткодействие
Аноним 10/02/16 Срд 14:41:59 #68 №260314 
14551045197690.png
>>259722
Хорошо.
Можно ли повысить эффективность ускорителя повышением напряжения до ебаной бесконечности, если между ускоряющими секциями поставить некую преграду для поглощения электронов, излучающихся при превышении напряжения работы выхода материала?
Ограничение на максимальную мощность электрического поля прикладываюмую к телу я не нашел. Более того, с каждым вылетевшим из металла электроном остальные начинают требовать больше энергии для преодоления потенциального барьера на поверхности металла из за появившегося нескомпенсированного заряда протона.
Аноним 10/02/16 Срд 19:06:23 #69 №260441 
Анон, объясни мне что такое порядок величины. Только доступным языком, а не как педивикия.
Аноним 10/02/16 Срд 19:18:23 #70 №260448 
>>260441
Порядок величини записується за степенями 10. Наприклад, порядок величини 1500 = 3, оскільки 1500 може бути записана у вигляді 1,5×103. Відмінності в порядку величини можуть бути виміряні за шкалою десятинних логарифмів в «десятках» (тобто множниках числа десять). [1] Числа різних порядків даються, наприклад, власними назвами степенів тисячі.

"Два числа мають той же порядок величини, якщо частка від ділення одного числа на інше менше 10. Наприклад, 23 і 82 мають той же порядок величини, але 23 і 820 - ні."[2] — Джон Баес[ru]
Аноним 10/02/16 Срд 19:24:40 #71 №260450 
>>260448
Блядь у меня глаза вытекли. Лучше бы ты это по-английски написал
Аноним 10/02/16 Срд 19:59:51 #72 №260471 
>>260450
Что, телевизор натаскал как пса ебучего?
Для тебя как для особенного.
Порядок это степень десятки необходимая для получению числа с такой же разрядностью как предложенное число.
Аноним 10/02/16 Срд 21:14:26 #73 №260514 
>>260314
в какомто смысле да
Если ты говоришь о количестве электронов
похожим образом работает фотэлектронный умножитель.

Если речь об энергии этого единственного электрона, то нет, она зависит тупо разницы их потенциалов этих электродов.
Но технически в импульстном режиме, это конечно улучшит установку, так как позволяет накопить побольше зарядя перед пробоем.

Металлические пластинки будут местом утекания электронов, такчто в этом моменте они будут ухудшать выход разогнанных электронов, и соотвественно общий кпд установки.

Позволят ли они прикладывать больший потенциал к аноду с кактодом, от пластинок и диэлектриков это мало зависит, это больше зависит от твоей способности накачивать электроны в катод(или отсасывать их из анода). Те зависит от способности создавать разность потенциалов. Грубо говоря от максимальной напруги на клеммах твоего источника питания, в котором вот тебе диэлектрики понадобятся в хороших количествах, чтобы повысить этот потенциал источника, до момента когда ему там все потроха начнет пробивать.

Относительно взаимодействия этой емкости (пластинок с проводниками и не проводниками) как она будет взаимодействовать с пучком электронов, не берусь судить, какие там будут наводки, и как она начнет протекать и как она начнет превращаться в соленоид и как получившиеся магнитные поля начнут влиять на электроны, но ни к чему хорошоему (относительно энергии ускоренных электронов) это пожалуй приводить не должно. Может ли там возникнуть какоето переменное магнитное поле, какаято автогенерация, которая поможет дополнительно разгонять электроны, за счет утекания энергии, вот это хз, но думаю если не постараться то нет.

Если у тебя эти пластинки служат для создания разгонного потениала, то это другой вопрос.
Аноним 10/02/16 Срд 21:22:24 #74 №260518 
>>260314
делай короче циклотрон и не еби мозги, деды смогли и ты смогешь, весь термояд твой. На ебее купишь запчасти, ну или самому из палок.

http://files.school-collection.edu.ru/dlrstore/428a1846-6e7d-16ce-71f6-2e8cdc140392/1002305A.htm
Аноним 10/02/16 Срд 21:56:51 #75 №260542 
>>260514
Ты, по моему, немного не понял.
Я говорю о ускорении пучка электронов/ионов напряжениями, которые превышают напряжения пробоя вакуума. А чтобы не было пробоя, ставить между ускоряющими секциями преграду которая будет ловить вылетешие электроны.

Я и не говорил что они ПОМОГУТ ПРИКЛАДЫВАТЬ большее напряжение, я спрашивал о том РАЗРЕШАТ ли они прикладывать болеше напряжение. Это и так понятно что энергия зависит от разности потенциалов между двумя секциями, а разность потенциалов можно приложить какую угодно большую. Ограничивает её только пробой вакуума.

Переформулирую еще раз, чтобы наверняка.
Есть две секции ускорителя. К ним прикладывается охуительно напряжение, например 20 мегавольт. Естественно они начинают излучать электроны во все стороны и может случится пробой. Чтобы его не допустить, я предлагаю поставить между секциями кольцевые пластинки с диаметром меньшим чем диаметр секций. Вот по тому промежутку внутри кольцевых пластинок может пробить или нет, я спрашиваю.

>>260518
У них кпд КРАЙНЕ МАЛ. У линейнх ускорителей кпд теоретически в районе 99.8%, а практически 90-95, в то время как у кицлотронов в лучшем случае 60%, и это не считая излучения частицы движущейся в наоднородном магнитном поле.
Аноним 10/02/16 Срд 22:13:19 #76 №260549 
>>260542
ээ как все криво
работай в стационарном режиме, тебе пробой и нужен, стационарный, потом наростишь напругу до скольки хотеть
Аноним 10/02/16 Срд 23:20:08 #77 №260588 
14551356083270.png
>>259700 (OP)
Аноны, объясните болвану!
Используют ли в жидкостных ракетных двигателях ТНА, приводимые в движение с помощью электодвигателя? Что будет, если включить такой ТНА без горючего, просто в воздухе? Даст ли он хоть какую-то тягу в двигателе тупо прокачивая через себя воздух?
Аноним 10/02/16 Срд 23:23:51 #78 №260589 
>>260114
>>260114
>>260135
>>260135
Спасибо, дорогой друг!
Аноним 10/02/16 Срд 23:28:41 #79 №260594 
>>260136
а Z - это шо, красное смещение чтоли?
Аноним 10/02/16 Срд 23:38:33 #80 №260603 
>>260588
Используют для мелкоракет с акумуляторами. Если включить, то будет хуйня с ТВР в тысячи раз меньше нормального режима работы. Абсолютно сосет у пропеллера с двигателем и источником питания того же веса.
Аноним 10/02/16 Срд 23:53:53 #81 №260610 
14551376337940.jpg
14551376338021.jpg
>>260588
Нахуя? Чем питать? Это охуеть как неэффективно же, удельная плотность хранения электричества КРАЙНЕ МАЛА.

Небольшую тягу даст конечно.

Если интересны альтернативные схемы - например есть насос на поршневых ДВС (!), впихнутый XCOR в движок Lynx в целях удешевления.
Аноним 10/02/16 Срд 23:57:45 #82 №260619 
14551378657650.png
>>260603
А если нужно обойтись только средствами самого ЖРД для его спасения и повторного испольования? Если допустить, что где-то там есть источник питания, способный выдать много энергии за маленькое время (вроде связки конденсаторов или хз что еще), имеет ли смысл использовать такой способ получения тяги, чтобы смягчить посадку?
Аноним 10/02/16 Срд 23:59:22 #83 №260622 
>>260610
>>260619
Это тоже сюда, пардон.
Аноним 11/02/16 Чтв 00:10:43 #84 №260633 
>>260619
Лол, нет конечно. Тяга хуйня будет.
> Если допустить, что где-то там есть источник питания, способный выдать много энергии за маленькое время
Штука в том, что весить он будет раз этак в 20 больше своего энергетического эквивалента в топливе. Бессмысленно.
Аноним 11/02/16 Чтв 00:15:40 #85 №260634 
>>260619
Проще петарды вешать, как у СА Союза.
Аноним 11/02/16 Чтв 00:21:42 #86 №260637 
>>260633
>>260634
Спасибо что объяснили, добра вам!
Аноним 11/02/16 Чтв 11:44:24 #87 №260712 
>>260549
Почему нужен?
Аноним 11/02/16 Чтв 12:28:50 #88 №260716 
14551829301750.jpg
Господа космонавты, кто следит, как там дела обстоят с предполагаемой 9-й планетой?
Аноним 11/02/16 Чтв 13:11:53 #89 №260722 
>>260716
Есть отдельный тред, но новостей нет. Пока они все небо не облазят и не поймут, что наебались ждите. На это уйдет от трех до 5 лет. Очередь на телескопы расписана на годы вперед.
https://2ch.hk/spc/res/252094.html
Аноним 11/02/16 Чтв 13:45:59 #90 №260731 
Какая плотность у чёрной дыры, пацаны, бля? Ниже, чем у начальной сингулярности или выше, сука? Нихуя не понимаю, блядь, чё за хуйня, блядь, сука.
Аноним 11/02/16 Чтв 13:49:00 #91 №260735 
>>260731
Ниже. Базарю.
Аноним 11/02/16 Чтв 13:50:50 #92 №260736 
>>260735
Бля, ну тогда пойду перепроверять свои расчёты. Сраный научрук мне всю плешь проел, пидор, блядь.
Аноним 11/02/16 Чтв 13:56:46 #93 №260739 
>>260731
Плотность - это масса разделить на объем. ρ=m/V Так что средняя плотность сверхмассивной черной дыры может быть не больше, чем у киселя. ЧД - это область пространства условная граница которой - горизонт событий.
В космологической же сингулярности сидела вся Вселенная целиком.
Аноним 11/02/16 Чтв 14:43:31 #94 №260757 
14551910120300.png
Вам конец, жалкие земляне!
sageАноним 11/02/16 Чтв 15:08:37 #95 №260763 
>>260757
Кто ходил в сайфай и опять забыл закрыть дверь?
Аноним 11/02/16 Чтв 15:12:38 #96 №260765 
>>260763
Ну я ходил.
Аноним 11/02/16 Чтв 15:13:46 #97 №260766 
>>260765
Нахуй ты вызвал... Нахуй ты говном вымазался?
Аноним 11/02/16 Чтв 15:16:12 #98 №260768 
>>260766
Так это... я же... вот в деревнях-то... в деревнях-то всё читали!
Аноним 11/02/16 Чтв 15:18:10 #99 №260769 
>>259700 (OP)
хуй
Аноним 11/02/16 Чтв 15:18:52 #100 №260770 
>>260769
ОП.

Теория Всего найдена.
Аноним 11/02/16 Чтв 17:27:37 #101 №260790 
Почему Роскосый это официально постит вконтакте ?

>На этой неделе в вечернее время суток траектория МКС проходит над большей частью Европейской территории России, а это значит, что появилась возможность не только увидеть станцию, но и сфотографировать ее.
МКС движется в направлении с запада на восток, увидеть самый яркий искусственный спутник Земли можно даже невооруженным глазом.
Ждем ваши фото и видео в комментариях под этим постом!
P.S.: к посту прикреплены графики пролета станции над несколькими крупными городами. Если вы не нашли там свой город, то перейдите на сайт www.heavens-above.com, выберите свое местоположение (панель в правом верхнем углу), а затем на главной странице укажите «Cпутники-МКС».

Я полистал сайтик и сделал вывод, что событие совсем рядовое. Происходит довольно часто. Чем отличается от обычных ?
Аноним 11/02/16 Чтв 17:40:46 #102 №260793 
>>260790
Ну надо-же чем-то бампать, вот одмин группы и вбрасывает.
Аноним 11/02/16 Чтв 18:07:38 #103 №260801 
>>260790
Да просто вбрасывают чтобы заинтересовать вк-аудиторию. Далеко не все в курсе, что МКС легко наблюдается глазами, или где и как ее искать вообще, а кто-то знал, но попалось на глаза. Вот видишь, ты же полистал, заинтересовался. Популяризация.
Аноним 11/02/16 Чтв 18:56:21 #104 №260822 
>>260790
>Почему Роскосый это официально постит вконтакте
Для этого:
>Я полистал сайтик и сделал вывод, что событие совсем рядовое.
Аноним 11/02/16 Чтв 19:02:36 #105 №260827 
С какой скоростью для Вселенной, если бы она была живой, то есть могла бы быть субъектом, проходило бы время? Ну, допустим, одна минута или миллион лет для неё были бы какой длительности?
Аноним 11/02/16 Чтв 20:01:08 #106 №260878 
>>260827
От чего так штырит?
Аноним 11/02/16 Чтв 22:10:37 #107 №260967 
>>260827
Есть мнение, что время разродилось в момент БВ, так что ни сколько.
Аноним 11/02/16 Чтв 23:40:14 #108 №261061 
>>260827
утром стала, по быстрому перепихнулась, накрасилась, на работу, вечером спать
такчто день, на следующее утро все по новой
Аноним 12/02/16 Птн 01:54:39 #109 №261137 
>>259700 (OP)
оп ты чето хуевую картинку воткнул, три раза промотал прежде чем нашел тред

по хардкору что это за хуита Black Knight Satellite
есть-нет, параметры орбиты, номер в каталге
Аноним 12/02/16 Птн 02:20:13 #110 №261145 
>>261137

>по хардкору что это за хуита Black Knight Satellite
Ничего. Фантазии, вроде баз на луне и палеоконтактов.
>есть-нет
нет
Аноним 12/02/16 Птн 02:29:34 #111 №261148 
>>261145
Я бы не был так категоричен. Black knight много раз наблюдался.
Аноним 12/02/16 Птн 02:33:11 #112 №261149 
http://mixstuff.ru/archives/89643
Аноним 12/02/16 Птн 02:38:40 #113 №261151 
>>261148
>много раз наблюдался.
В пятницу вечером, после второй поллитра.
В зог пиздуй, со своими охуительными историями
Аноним 12/02/16 Птн 02:40:11 #114 №261152 
14552340116820.jpg
>>261148
Ага, поэтому в качестве пруфа повсюду только снимок куска теплоизоляции с миссии STS-88?
Аноним 12/02/16 Птн 02:56:53 #115 №261155 
>>261137
Помню в детстве читал книжку, там черным принцем называлась АМС с погибшей планеты, куда они запихали более продвинутый аналог наших пластин и другой хуйни. Кажется автор Казанцев. Это популярная байка времен космогонки, только и всего, ее эксплуатировали все, от журналюг до пейсателей. А фотка просто мусор. Спутник якобы летает с востока на запад, хуй ты его сфоткаешь с МКС.
Аноним 12/02/16 Птн 03:46:08 #116 №261163 
>>261145
>>261149
kk

печалька, я уже было подумалтолько что компанию на кикстартере мутить, это всяко веселее чем проверка лунной темы
Аноним 12/02/16 Птн 10:55:14 #117 №261184 
>>260878
От глицина.
>>260967
Хм… звучит правдоподобно.
Аноним 12/02/16 Птн 13:20:28 #118 №261231 
>>260739
Только вот объём чд штука крайне сложная.
Аноним 12/02/16 Птн 13:27:20 #119 №261240 
Что будет если земля мгновенно перестанет крутиься вокруг своей оси?
Аноним 12/02/16 Птн 13:30:44 #120 №261246 
>>261240
всех снесет к й матери, по правте если не прав
Аноним 12/02/16 Птн 13:57:27 #121 №261260 
>>261246
Ага, полетишь вперед по инерции с ебической скоростью и убьешься о ближайшую монолитную конструкцию.
Аноним 12/02/16 Птн 13:58:59 #122 №261263 
>>261231
Да нет там нихуя сложного, горизонт событий зависит только от массы. А в контексте того вопроса вообще всеми погрешностями можно пренебречь.
Аноним 12/02/16 Птн 14:16:09 #123 №261268 
Sup, спейсач.
Накидай пару охуенных научно-популярных (и красивых) фильмов о космосе. С меня как обычно.
Аноним 12/02/16 Птн 14:19:54 #124 №261269 
>>261268
Интерстеллер же
Аноним 12/02/16 Птн 14:24:31 #125 №261271 
>>261268
Самый крутой и красивый на текущий момент
Космос: пространство и время
Аноним 12/02/16 Птн 14:37:52 #126 №261273 
14552770725110.jpg
>>261269
Слишком крут, чтобы смотреть.
>>261271
>Космос: пространство и время
Спасибо! Держи няшку
Аноним 12/02/16 Птн 14:47:11 #127 №261274 
14552776314100.jpg
Ответьте дибилу, который заебался читать желтуху об экспедиции на Марс. На сегодняшний момент, когда запланирована первая экспедиция на красную планету, сколько человек в ней участвует и сколько она должна длиться по времени?
Аноним 12/02/16 Птн 14:53:39 #128 №261276 
>>261274
Никогда
Ноль
Ноль
Аноним 12/02/16 Птн 14:59:07 #129 №261279 
>>261276
что все так хуево?
Аноним 12/02/16 Птн 15:19:31 #130 №261284 
>>261274
Приходи лет через 10. Может там уже родят что-нибудь определенное.
Аноним 12/02/16 Птн 15:37:47 #131 №261291 
>>261279
жди сороковые
Аноним 12/02/16 Птн 15:54:40 #132 №261295 
14552816806740.jpg
>>259700 (OP)
Как коррелирует спин электрона с сингулярностью черной дыры?
Аноним 12/02/16 Птн 15:59:45 #133 №261302 
>>261260
>монолитную конструкцию.
Есть подозрение, что монолитная конструкция вместе с несколькими метрами грунта тоже полетит.
Аноним 12/02/16 Птн 16:06:23 #134 №261306 
14552823831790.jpg
>>261274
На текущий момент - на конец тридцатых. То есть никогда.

>>261279
Любой проект, который потребует больше пяти лет, никогда не завершится.
Аноним 12/02/16 Птн 16:24:20 #135 №261313 
>>261263
Сложность в том, как стоит считать радиус чд - саму сингулярность или же вместе с окружающий её хуитой. Поэтому плотность чд по идее близка к бесконечности ну или как там еще плотно упаковать можно.
Аноним 12/02/16 Птн 16:31:01 #136 №261316 
>>261313
>Сложность в том, как стоит считать радиус чд
Ты что, идиот? Определение радиуса Шварцшильда сам найти сможешь?
Аноним 12/02/16 Птн 16:34:25 #137 №261319 
>>261316
Могу. Ты действительно считаешь радиус чд по радиусу Шварцшильда?
Аноним 12/02/16 Птн 16:47:26 #138 №261323 
>>261319
Хм, ну черная дыра это как бы область пространства, ограниченная этим радиусом. Это определение. Что ты там считаешь - уже другой вопрос.
Аноним 12/02/16 Птн 17:09:52 #139 №261329 
>>261323
Я считаю что ты ебучий дегенерат, но не это суть.
Аноним 12/02/16 Птн 17:15:16 #140 №261330 
>>261329
Какой весомый аргумент.
Аноним 12/02/16 Птн 17:26:45 #141 №261334 
>>261330
Весомые аргументы находятся в умных книжках, например Новикова - физика чёрных дыр, но тупые дегенераты вроде тебя предпочитают читать паблики Вконтакте для таких же тупых дегенератов.
Аноним 12/02/16 Птн 17:34:49 #142 №261339 
>>261334
Бешеное животное, зарепортил тебя. Кончено же модеру похуй, но просто захотелось, чтоб ты знал.
Аноним 12/02/16 Птн 17:41:37 #143 №261343 
14552880976570.png
>>261334
Зачем ты срешь себе на голову, дегенерат? Вот глава из этой книги.
Аноним 12/02/16 Птн 17:49:14 #144 №261348 
>>261343
Тупой дегенерат не понимает за что его называют тупым дегенератом. Тупой дегенерат не читал ВСЮ книгу и не понимает о чем там речь.

У тебя какая то особенная потребность в самоунижении, а?
Аноним 12/02/16 Птн 18:00:31 #145 №261352 
13212
Аноним 12/02/16 Птн 18:05:46 #146 №261353 
>>261348
Не, петушок, так не пойдет. Вот книга Новикова, он точно дает определение в ней черной дыры, или ты хочешь меня убедить в том, что где-то через 10 страниц там будет написано, что это все хуйня и он пошутил?
Алсо, если ты прочитал одну книгу 1986 года одного автора, то ты молодец, конечно, хоть что-то прочитал, но это вовсе не значит, что ты стал охуенным специалистом.
Аноним 12/02/16 Птн 18:10:10 #147 №261357 
>>261353
Это тебе не паблики листать, дегенерат.
Аноним 12/02/16 Птн 18:15:36 #148 №261359 
>>261357
Ты апеллировал к книге, цитата из книги приведена. В книге черным по белому написано, что радиус ЧД ограничен горизонтом событий, дальше ты срываешься на визг и оскорбления.
Слив засчитан, я полагаю.
Аноним 12/02/16 Птн 18:40:09 #149 №261370 
14552916095560.png
>>261359
Я апеллировал к тому, что ты тупой дегенерат, который нихуя не понимает в теме ни-ху-я-ше-чки.

Пикрелейтед - примерный матан по которому считаются требуетмые величины, а не та блевотина, которую ты притащил в мой ламповый итт тред с вконтакта, википедии или откуда вы там долбаёбы лезете и самоотверженно, с рвением тупого долбаёба раз за разом просишь ссать тебе прямо на голову, долбаёб.

>Слив засчитан, я полагаю.
ох лол
Аноним 12/02/16 Птн 18:52:01 #150 №261374 
>>261370
Какие требуемые величины, че несешь вообще? Ты начал хуйню городить, отвергая горизонт событий как границу ЧД, хотя во любой литературе черной дырой называют область пространства, ограниченную горизонтом событий. К плотности материи под горизонтом это никакого отношения не имеет.
Аноним 12/02/16 Птн 18:56:12 #151 №261376 
>>261374
Я ничего не отвергал, тупой ты идиот. Изначально претензии были, к тому что брать за радиус при подсчете плотности для чувака вопросом выше.
Аноним 12/02/16 Птн 19:06:07 #152 №261384 
>>261370
во, и сразу все понятно, так и надо
Этот тред освящен высшим существом
Аноним 12/02/16 Птн 19:12:56 #153 №261390 
>>261376
>Я ничего не отвергал
>как стоит считать радиус чд - саму сингулярность или же вместе с окружающий её хуитой
>Приводит уравнения в которых радиус ЧД высчитывается с учетом горизонта событий.

Петух продолжает вертеть сракой, найс.
Аноним 12/02/16 Птн 19:25:17 #154 №261409 
>>261390
А что не так, ебанько? Или ты тоже жиденько обделаться хочешь в споре самим Иисусом теор. Физики?
Аноним 12/02/16 Птн 19:40:29 #155 №261416 
>>261409
>в споре самим Иисусом теор. Физики?
Петух запостил скриншот из книги, даже не поняв, что там нарисовано. Для него эти формулы не больше чем священные каракули на стенах какого-нибудь храма имени Мумбы-Юмбы.
>А что не так
Я написал что не так. Петух копротивлялся за то как считать радиус ЧД: саму сингулярность, или с горизонтом событий, изошелся на визг и притащил формулы в которой учитывается горизонт событий, и ты спрашиваешь, где он обдристался? Да везде.
Аноним 12/02/16 Птн 19:54:51 #156 №261441 
>>260822
Я то думал, что сейчас происходит какая-то особенная спутниковая конфигурация, особенно яркая и надо обязательно пойти ее зафотать!
То есть, нет ? Все как обычно ?
Аноним 12/02/16 Птн 20:24:06 #157 №261465 
>>261441
Нет никаких особенных спутниковых конфигураций, просто в некоторых случаях МКС наблюдается, а в некоторых нет.
Аноним 12/02/16 Птн 21:26:19 #158 №261478 
Шкальник-гуманитарий врывается в тред.
Возможно ли существование на Марсе хоть каких-то земных организмов без скафандра? Да, там дубак, но в Антарктиде тоже есть какие-то бактерии, лишайники всякие. Возможна ли такая хуйня: транспортировка их в марсоходе, марсоход ездит-ездит, детектит наличие воды (ведь она там есть?), подсаживает туда организмы. Или там все токсично?
Аноним 12/02/16 Птн 21:40:52 #159 №261482 
>>261478
Предположительно да, возможно. Существуют даже запреты привнесения земных организмов, обеззараживают марсоходы и все такое.
Аноним 12/02/16 Птн 21:43:40 #160 №261483 
>>261478
>Да, там дубак, но в Антарктиде
В антарктиде давление атмосферы позволяет твоей тушке жить. На Марсе у тебя газы в крови закипят.

Ну, и еще там нечем дышать. И радиация всеразличная. Ну и холодновато. Так что без жбана-скафандра я бы не рекомендовал.
Аноним 12/02/16 Птн 21:43:41 #161 №261484 
>>261482
Боятся эффекта кроликов в Австралии?
Аноним 12/02/16 Птн 21:44:50 #162 №261485 
>>261483
Про человека даже не спрашиваю. Я про бактерии. Максимум лишайники какие-нить морозостойкие-антарктические. Или на них это тоже влияет?
Аноним 12/02/16 Птн 21:49:58 #163 №261489 
>>261478
Подземные воды и пещерки - там вполне может, что то быть.
Аноним 12/02/16 Птн 21:50:15 #164 №261490 
>>261484
Да, собственно почва там вполне. Единственная проблема температура, но чем черт не шутит. Другой вопрос почему на марсе нет растений, если по имеющимся данным условия есть?
Аноним 12/02/16 Птн 21:52:24 #165 №261493 
>>261478
Да, тут не так давно ученые ссали кипятком, мол НАСА могло Марс заразить земной жизнью.
http://phys.org/news/2014-05-bacteria-species-curiosity-baggage-mars.html
А при подготовке марсохода к отправке вроде как немного обосрались со стерилизацией.
Аноним 12/02/16 Птн 21:56:28 #166 №261494 
>>261485
>Про человека даже не спрашиваю. Я про бактерии. Максимум лишайники какие-нить морозостойкие-антарктические. Или на них это тоже влияет?
Биосфера же не из одного вида состоит. Организму (бактерии/лишайнику) надо откуда-то энергию брать, иначе это уже не живой организм. А это уже перенесение на Марс целых экосистем. Которые, с большой вероятностью, там существовать не смогут.

Конечно можно помоделировать перенесение туда какого-нибудь вида-одиночки (хемосинтетика?), но это просто фантазии. На Земле-то трудно предсказать, а такую модель для Марса строить - не хватит ни сил, ни данных.
Аноним 12/02/16 Птн 21:58:07 #167 №261500 
>>261494
Тогда бредни НАСЫ и обеззараживание аппаратов - хуйня?
Аноним 12/02/16 Птн 22:01:23 #168 №261509 
>>261490
На Земле жизнь появилась в достаточно теплом океане и только потом развилась во всем своем многообразии. Может, экосистема не может стартануть сразу с морозостойких бактерий?
Аноним 12/02/16 Птн 22:03:04 #169 №261511 
>>261509
Это если стартануть. А вот сохраниться может, не находишь? Пока однозначного опровержения нормальной жизни на Марсе в прошлом при атмосфере не было.
Аноним 12/02/16 Птн 22:03:36 #170 №261512 
>>261494
А что нужно бактериям, кроме воды и, возможно, света? Даже кислород в таком количестве появился только благодаря им, а не они от него.
Аноним 12/02/16 Птн 22:10:07 #171 №261518 
Там пиндосы гравитационные волны нашли. Обсудили уже? Тред не нашел.
Аноним 12/02/16 Птн 22:19:56 #172 №261530 
>>261518
Ебать ты пиздоглазый
https://2ch.hk/spc/res/259962.html
Аноним 13/02/16 Суб 09:11:03 #173 №261660 
14553438636320.jpg
>>261530
Аноним 13/02/16 Суб 10:46:42 #174 №261671 
Анон, посоветуй книжек о дальнем космосе новичку:
Что почитать о дальних звёздах, чёрных дырах, других галактиках?
Что почитать о Солнечной системе, её планетах, спутниках этих планет?
Каких авторов и какие книги рекомендуешь? Только не худлит, а научную или научно-популярную литературу.
Пока что я читал только Википедию и Лурку на эту тему
Аноним 13/02/16 Суб 17:21:04 #175 №261784 
Спейсач, у меня встал вопрос.
Если например в США получили новую инфу или снимки из космоса могли ли передать их в распоряжение Союзу и наоборот.
И как с этим обстоят дела сейчас.
Аноним 13/02/16 Суб 17:33:38 #176 №261792 
>>261784
Даже больше того - международные инструменты летали. Научный обмен всегда был, фундаментальная наука вроде планетологии всегда интернациональна и ей похуй на железные занавесы. бен_рич_и_уфимцев.jpg
Аноним 13/02/16 Суб 17:44:52 #177 №261797 
>>261792
Тогда тут другой вопрос, почему не было совместных проектов, а имела место сраная гонка. Вместе же эффективнее, а АМС, например, каждый запускал свои.
Аноним 13/02/16 Суб 17:50:19 #178 №261799 
>>261797
>почему не было совместных проектов
Научных? Были же. За ноу-хау конечно грызня стояла, она всегда была и есть.
>Вместе же эффективнее
Абсолютно не факт.
Аноним 13/02/16 Суб 17:53:37 #179 №261801 
>>261799
>Абсолютно не факт
Ну во всяком случае, было бы больше трудовых и материальных ресурсов и умственных
Аноним 13/02/16 Суб 17:57:53 #180 №261808 
>>261801
требовалась бы хорошая система координации, куча доброй воли, общее видение того что хочется получить.
А одни хотят денег, а другие хотят сидеть на трубах, и пока они договорятся другие уже за горизонт уплывут.
Аноним 13/02/16 Суб 17:58:10 #181 №261809 
>>261801
И не было бы конкуренции. Но хрен с ней, самое главное что организация всего этого, тем более через границы, сложнее на несколько порядков, это не преувеличение. Это слабая сторона любых многонациональных проектов. Прорывы чаще всего делают именно маленькие коллективы за счет организационной простоты (skunk works, советские ОКБ, и т.п.) Не может сложная стуктура породить простой и ясный продукт, принципиально.
Аноним 13/02/16 Суб 18:00:26 #182 №261811 
>>261808
>>261809
Понятно, спасибо за объяснение
Аноним 13/02/16 Суб 21:35:02 #183 №261909 
>>259700 (OP)
>>329318
Итак, поясните насчет давление света на зеркальную поверхность (будем считать, что она отражает 100%).

Конкретно в статье ниже, которую я скинул, указано, цитирую: "отразившись от зеркала паруса, луч меняет импульс в два раза". И далее приведенная формула. Непонятный момент - зеркало получает двойной импульс? И как вообще свет оказывает давление на идеальное зеркало, если оно полностью отражает фотоны? Свет должен терять энергию на давление (тягу) и тем самым частота его должна уменьшаться?

http://go2starss.narod.ru/pub/E002_LSIP.html
Аноним 13/02/16 Суб 22:26:59 #184 №261937 
>>261909
>И как вообще свет оказывает давление на идеальное зеркало, если оно полностью отражает фотоны?
поглощает, отражает - это неупругое и упругое столкновение, никакой разницы из-за того что это свет, нет.

>Свет должен терять энергию на давление (тягу) и тем самым частота его должна уменьшаться?
да
Аноним 13/02/16 Суб 22:44:54 #185 №261945 
>>261937
>да
Спасибо, значит, я просто неправильно понял статью.

Не могу еще понять вот эту фразу:
"При падении света на зеркальную поверхность удар фотона считают абсолютно упругим, поэтому изменение импульса и давление в 2 раза больше, чем при падении на черную поверхность (удар неупругий)."

Т.е. упругий удар об зеркало создает в два раза больше давления, чем неупругий об абсолютно черное тело? Это связано с тем, что зеркало, помимо получения энергии фотона, еще и излучает его как массу обратно?
sageАноним 13/02/16 Суб 22:47:46 #186 №261946 
>>261945
>Это связано с тем, что зеркало, помимо получения энергии фотона, еще и излучает его как массу обратно?
Советую к прочтению книгу Фейнмана "КЭД" там довольно доступно объяснено каким образом фотоны отражаются от зеркала и почему так происходит.
Аноним 14/02/16 Вск 00:24:27 #187 №262013 
>>261946
Спасибо, буду просвещаться
Аноним 14/02/16 Вск 00:56:23 #188 №262036 
>>261945
>да
>Спасибо, значит, я просто неправильно понял статью.
Не ходил по ссылке, не в курсе что там написано, просто ты правильно подумал. Другое дело, что в обсуждениях это могут опускать, особенно когда обсуждают только давление и потенциальное ускорение крафта, и вот по какой причине.
Причина в ускорении крафта и скорость крафта(на начальном этапе по крайней мере), которое не велико, относительно скорости света.
То о чем ты говоришь, это доплеровское смещение(wiki/Эффект_Доплера), используется радарными установками для определения скорости целей, к примеру.
Если крафт ускоряется допустим 1м/с, то достаточно продолжительное время величина этого эффекта небольшая, хотя и детектируемая.

по статьесходил всетаки, но сломался:
>Принимая стоимость одного киловатта электроэнергии по цене 2$
0.1 бакса киловатт, но кпд лазера превращает его в 1-2 бакса ок.
>Очевидно, что стоимость энергии для разгона даже маленького межзвездного зонда будет огромной.
Собственно поэтому смещение частоты не высокое, вообщем это две стороны одного и того же, что импульс не есть энергия, производная от, но не одно и тоже.

>давление в 2 раза больше
Сам по себе импульс это векторная величина, и импульсы можно складывать как векторы.
И есть закон сохранения импульса.
К примеру, если в системе два тела, одно покоится, а другое налетает на него.
То у первого тела импульс равен нулю, а у другого равен 1(к примеру), то не зависимо от столкновения, упругое или нет, суммарный импульс тел должен быть равен 1.
Раскололось оно на 20тыщь кусков, которые, каждый из них, полетел в свою сторону(и у каждой скорость овер9000, суммарная кинетическая энергия системы при этом изменилась, а суммарный импульс нет), или мы включили супер отталкивающий магнит и тд.
Суммарный момент всех частей системы будет равен 1.

Фотон поглотился болванкой, и переизлучился во все стороны(!) в виде теплового излучения(в этом случае импульс от теплового излучения будет равен нулю), то болванка должна иметь итоговый импульс того фотона который она поглотила, а если фотон полетел обратно, то для того чтобы общий импульс системы не изменился болванке придется иметь 2 импульса (ну это как 0 + 1 = 1 + 0 = 2 + -1 = 3 + -2. импульс фотона был сначала 1, а потом стал -0.99999999(примерно -1)

кхм, короче гугли закон сохранения импульса, и что такое импульс, прикол в этом.
Аноним 14/02/16 Вск 01:07:58 #189 №262042 
>>261945
>>261946
и еще раз, не важно что это фотон, не важен механизм отражения
важно сохранение импульса системы, по крайней мере на уровне это статейки
которая в принципе норм, но все еще проще и лучше чем в ней написано.
Аноним 14/02/16 Вск 02:47:36 #190 №262077 
>>262036
>Фотон поглотился болванкой, и переизлучился во все стороны(!) в виде теплового излучения(в этом случае импульс от теплового излучения будет равен нулю), то болванка должна иметь итоговый импульс того фотона который она поглотила, а если фотон полетел обратно, то для того чтобы общий импульс системы не изменился болванке придется иметь 2 импульса (ну это как 0 + 1 = 1 + 0 = 2 + -1 = 3 + -2. импульс фотона был сначала 1, а потом стал -0.99999999(примерно -1)

Да, именно об этом я думал, вот теперь все ясно :) Зеркало испускает фотон обратно и приобретает его же импульс еще раз, вследствие чего свет оказывает на идеальное зеркало в два раза большее давление, чем на абсолютно черное тело.

>0.1 бакса киловатт, но кпд лазера превращает его в 1-2 бакса ок.
Статья старая и, похоже, расчеты автор делал чисто для себя еще по ценам тех времен.
Аноним 14/02/16 Вск 04:49:01 #191 №262122 
>>262077
Цены не сильно изменились, просто они сразу посчитали с учетом кпд лазера.(который они взяли походу за 10 процентов)
Хотя такой подсчет и дает цифру, которую можно пощупать, он бессмысленен в плане оценки.
Так как стоимость киловатта, полученного в космосе а не на земле, может быть как существенно дороже так и существенно дешевле.

Мое мнение, что если есть возможность вывести или сделать необходимое оборудование, для постройки 1000 км-ровой линзы, за разумную цену, то килловат существенно существенно дешевле земного.
Было желание откоментить статью вдоль и поперек, но чето не поднялась рука.

на может понравится http://www.swinburne.edu.au/news/latest-news/2016/01/focus-on-results.php
Аноним 14/02/16 Вск 06:44:13 #192 №262149 
14554214535590.jpg
Блядь, смотрел вчера говнодокументалку. И там рассказывалось, что к Солнечной системе движется звезда Глизе 710, и что через миллион лет где-то она долетит и своей гравитацией сделает облаку Оорта твой дом труба шатал. И после этого оттуда полетят кометы в охуительном количестве и земляшке, вернее жизни на ней, пиздарики. Это так?
Аноним 14/02/16 Вск 07:02:58 #193 №262150 DELETED
>>262149
> и земляшке, вернее жизни на ней, пиздарики. Это так?
> через миллион лет
А оно тебе сейчас жить мешает? Ты понимаешь, что если человечество не выберется с земляшки в ближайшие сто лет, то то, отчего погибнет человеческая цивилизация - не будет иметь большого значения. От кометы даже лучше - вроде как и не сами себе галактическую премию даврина сделали, и почти без мучений. Только ждать долго.

Апатичная цивилизация, ни капли не жаль.
Аноним 14/02/16 Вск 08:32:12 #194 №262155 
14554279327570.jpg
>>262150
Нет, ну просто я столько слышал, что Солнце распухнет и зажарит Земляшку, поэтому полюбасу надо съебывать с планеты. Но это будет через сотни миллионов лет, так что можно не париться и подыскивать на расслабоне себе теплое местечко.
А тут, блядь, оказывается, что в запасе у нас нихуя не сотни, а один всего миллион хотя и это дохуища просто времени, да, согласен
Аноним 14/02/16 Вск 09:42:06 #195 №262166 
>>262149
Такие близкие прохождения случались и раньше. Например, около 10 млн. лет назад рядом прошмыгнула Альголь. Не так близко, в 8 св. годах, но Альголь намного масивнее и нет никаких катастрофичных импактных находок, связанных с этим событием. Алсо, оценки ущерба от прохождения Глизе 710 сильно разнятся. В документалках любят народ попугать, так кино более захватывающим получается.
Аноним 14/02/16 Вск 10:17:18 #196 №262170 
14554342382440.jpg
>>262149
По спейсачу такая вот работа гуляет и всплывает то там то тут.
http://iopscience.iop.org/article/10.1088/2041-8205/800/1/L17
70 тыс лет назад прямо через солнечную систему пролетела звезда Шольца, двойная. Пролетела в 38-75 тыс а.е., это через внешнее облако Оорта. Но вероятность ее проникновения во внутреннее облако Оорта (из которого как раз долгопериодические кометы и берутся, по представлениям) составляет 10-4.

Алсо, они ссылаются на работы, оценивающие что такие пролеты бывают довольно часто, в районе 10 штук на миллион лет, плюс-минус. Как видишь, пока все нормально. И делают вывод, что долгопериодические кометы могут прилетать от таких пролетов, а вовсе не из гипотетического облака Оорта. Кстати прилететь им оттуда тоже нужны тысячи, если не десятки тысяч, лет.

>земляшке
Если допустить все остальное (включая то, что облако Оорта вообще существует), полетят такие скорее всего не на Земляшку - если их кто и счистит, то дедуля Юпитер, единственная значимая масса кроме Солнца в нашей системе.

Короче, Gaia >>95979 (OP) должна замапить кучу звезд, и дать более точные ответы на вопросы о близких пролетах мимозвезд.
Аноним 14/02/16 Вск 12:51:23 #197 №262222 DELETED
>>262166
> рядом прошмыгнула
На два пальца левее.

Между прочим, может я конечно и хуй и все такое ящитаю, эти теории про возмущенное облако не особо охуенными. Ну то есть конечно, отрицать бессмысленно, но что мешает Альголю Шольцу или Барнарду иметь свои собственные облака тамошних оортов. И которые как раз краями догоняют внутренние планеты солнечки?
Аноним 14/02/16 Вск 13:16:45 #198 №262229 
>>262222
Ничего не мешает, но факты - вещь упрямая. Нет следов крупных катастроф, значит не так страшен черт, как его малюют.
Аноним 14/02/16 Вск 14:22:56 #199 №262243 
14554489768270.png
Важны ли будут наследственные болезни для будущих колонизаторов космоса?

Недавно узнал, что у меня есть одна такая болячка, которая может передаваться по наследству и понял, что мои дети тоже будут этим болеть. А значит ли это, что им могут закрыть путь в марсианские колонии? Представьте если не только мои зашкваренные гены окажутся там, но и всяких спидозников, олигофренов, гемофиликов, обладателей синдромов разных видов. На Марсе и так шанс мутаций гигантский, так еще и наш генетический балласт может туда попасть. А если дети там будут рождать, то и воспроизводиться.
Аноним 14/02/16 Вск 14:48:41 #200 №262265 
>>262243
>гены спидозников
Проиграл с этого адепта евгеники
Аноним 14/02/16 Вск 14:53:29 #201 №262268 
>>262243
>Важны ли будут наследственные болезни для будущих колонизаторов космоса?

>Недавно узнал, что у меня есть одна
>А значит ли это, что им могут закрыть путь в марсианские колонии?
К тому времени, когда/если будут марсианские колонии медицина будет способна править генетический код и блокировать такие гены. Вообще мы сейчас к этому в 1000 раз ближе, чем к марсианским колониям.
Аноним 14/02/16 Вск 14:53:43 #202 №262271 
>>262265
Ну, без евгеники никак. Мы реально можем построить общество без наследственных заболеваний на другой планете. Конечно радиация все равно будет воздействовать на колонистов, создавая новые наследственные заболевания, но мы можем отсечь наиболее опасные из тех, которые появились на Земле.

Кроме того мы можем полностью избавиться от различных паразитов и болезней, которые распространяются на Земли.
Аноним 14/02/16 Вск 14:57:53 #203 №262275 
>>262271 вот тут ответ по существу.
>>262268
Аноним 14/02/16 Вск 15:08:27 #204 №262283 
>>262268
К каждой отдельной болезни потребуется особый подход который растянет процесс борьбы с этими болезнями на века. Ты думаешь что появится некий метод-панацея, который позволит бороться со всеми болезнями сразу? это очень наивный взгляд на проблему.
Аноним 14/02/16 Вск 15:24:49 #205 №262290 
>>262283
Наивный взгляд на проблему - это думать что существует дохулиард генетических болезней, гены которых не локализованы, или не будут локализованы к тому времени.
Аноним 14/02/16 Вск 18:36:44 #206 №262330 
>>262268
>править генетический код и блокировать такие гены. Вообще мы сейчас к этому в 1000 раз ближе, чем к марсианским колониям.
Медик врывается в тред.
Дальше, а не ближе. На Марс минимум роверы забрасывали, а с геномом все очень плохо. Ты сейчас не можешь даже поменять цвет глаз (своих\эмбриона\кого хочешь). Какие уж там правки генетических уязвимостей. Так что правка генома сейчас на более раннем этапе, если сравнивать с колонизацией планет. Ну если это вообще можно сравнивать.
Аноним 14/02/16 Вск 19:20:10 #207 №262341 
>>262229
ну на счет нет это ты погорячился, я не к том что есть, а к тому что мы могли и не свзать одно с другим, в силу недостаточности информации.
Можно будет по луне судить, но для этого надо будет классифицировать кратеры на луне, по составу и тд. Тогда можно будет более точно судить что и как.
Церера тоже там можно будет датирвать обломки. Вообщем полнота ответа еще только предстоит.

>>262330
вот этого поддержу
Аноним 14/02/16 Вск 19:28:40 #208 №262344 
>>262330
Для Марса дохуя (очень, очень дохуя) денег надо, и никакой серьезной мотивации, кроме исследовательской, нет, чтобы их вкладывать. В этом плане медицина действительно ближе и актуальнее. Даже если для Марса у нас все уже есть, а медицина - все еще в глубоком развитии, все равно Марс остается гораздо, гораздо дальше. Так что тот анон прав.
Аноним 14/02/16 Вск 19:38:06 #209 №262347 
>>262344
>медицина действительно ближе и актуальнее
И как эта актуальность помогает в борьбе с раком и СПИДом? Лихорадкой Зика?

И даже если лечение появится, то все равно нужно будет проходить диагностику на эти заболевания на Земле, потому что лечить генетические заболевания на Марсе никто не будет. Опять же - скорее всего можно будет лечить новорожденного, а не взрослого больного.
Аноним 14/02/16 Вск 20:00:38 #210 №262355 
>>262347
К тому мы времени, как начнем жить на Марсе, проблем с медициной и генетикой будет минимум. Почему? Да потому, что это один из многих столпов, который, как и ресурсы, надо эффективно освоить и сделать максимально доступным, прежде чем куда-то лететь.
Ей богу, ты мыслишь так, как будто мы на Марс в ближайшие сотню лет переселимся. Если и полетят, то чисто как пиар-компания НАСА и т.д, в виде исключения. Колонизация Марса так же близка до нас, как колонизация соседней звезды, разве что чуть-чуть ближе. Генетика и исследования - это лишь вопрос времени. Это качественный подъем одной отрасли, в то время как полет на Марс требует качественного подъема всех отраслей, включая удешевления стоимости выхода в космос на килограмм груза, эффективности топлива, защиты от радиации, постройки промежуточной космической станции, способной обслуживать межпланетные корабли, постройки того самого межпланетного корабля и многое, многое другое.
Аноним 15/02/16 Пнд 10:24:46 #211 №262529 
Могут ли из черной дыры вылетать нейтрино?
Аноним 15/02/16 Пнд 10:50:56 #212 №262531 
>>262529
нет
Аноним 15/02/16 Пнд 10:54:35 #213 №262532 
>>262529
Да
Аноним 15/02/16 Пнд 11:29:41 #214 №262535 
>>262529
да нет вроде бы
Аноним 15/02/16 Пнд 12:01:52 #215 №262537 
>>262529
Нет, не могут. Нейтрино не двигаются со сверхсветовой скоростью.
Аноним 15/02/16 Пнд 12:20:24 #216 №262539 
>>262537
А почему тогда этот хуй, Нил Тайсон, в своей передачи про вселенную говорит, что во время большего взрыва когда вселенная еще была размером с мячик для пинг-понга и невероятной плотности, такой, что не выпускала свет от взрыва, пучки нейтрино уже вылетели за пределы этой быстро расширяющийся вселенной?
Аноним 15/02/16 Пнд 12:20:56 #217 №262540 
>>262529
А смысл им вылетать?
Аноним 15/02/16 Пнд 12:45:31 #218 №262546 
>>262539
Размером с мячик от пинпонга- это уже ебись какая огромная Вселенная, по сравнению с изначальной конфигурацией, при том стремительно расшеряющеяся. И вообще давай педерачу сюда, и таймкод, где он это говорил.
Аноним 15/02/16 Пнд 15:24:51 #219 №262564 
>>262539
>этот хуй, Нил Тайсон
ты сам ответил на свой вопрос лол
Аноним 15/02/16 Пнд 21:45:20 #220 №262648 
14555619209640.jpg
Блядь, анон посмотрел марсианина и разбил все лицо себе ладошкой от фейспалмов. Это же все ирл не пройдет, так ведь? Это бесконечное топливо когда корабль по два раза туда-сюда летает без дозаправки, этот прокол перчатки, картоха в мертвой марсианской почве, блядский брезент вместе обтекателя, это же все из пальца высосанное, так ведь?
Аноним 15/02/16 Пнд 21:48:56 #221 №262651 
>>262648
Конечно нет. Вот небольшая лекция кота на эту тему
https://www.youtube.com/watch?v=TfPh5aaMFRM
Аноним 15/02/16 Пнд 21:49:35 #222 №262652 
>>262648
Короче все там сферично и в вакууме. Прост у всех действующих лиц прокачана УДАЧА
Аноним 15/02/16 Пнд 21:53:48 #223 №262653 
>>262652
Ну, смотри - через месяц они выпустят книгу Science of the Martian, и люди прочитавшие книгу будут уверять нас что в фильме всё научно, что даже НАСА помогала со съемками фильма и что в книге СПЕЦИАЛЬНО ДЛЯ ТУПЫХ(для нас то бишь) указано, где есть просто отступления от науки для развития сюжета.
Все как с порашным Интерстелларом, который нам запрещают критиковать, потому что там якобы черную дыру на заднем фоне нарисовали с участием этого физика-теоретика. И потому что физика кротовых нор еще не разработана, но и не опровергнута, а значит возможна, а значит их можно лепить куда угодно и их критиковать также нельзя.
Аноним 15/02/16 Пнд 21:59:34 #224 №262655 
>>262651
>>262652
>>262653
А вот еще есть "Гравитация" фильм, там такой же ненаучный пиздей?
Аноним 15/02/16 Пнд 21:59:47 #225 №262656 
14555627874950.png
>>262648
Ну составные моменты там довольно тверды, плюс-минус сапог. Но всю книгу не покидало ощущение растянутой охуительной истории от пахома, да. Чирей как рычаг, ногу отрезали и обратно пришили, малафья полилася. на моменте бунта на корабле я дропнул нахуй, не выдержал

В целом что книга что фильм - терминальная стадия твердоты головного мозга. Дальше только стояние на одной ноге и курлык курлык.
Аноним 15/02/16 Пнд 22:01:36 #226 №262657 
>>262655
там хуже.
https://www.youtube.com/watch?v=ADrz5l_2eRo
Аноним 15/02/16 Пнд 22:03:08 #227 №262658 
>>262653
А вот на Интерстеллар не гони. Там сплошное гонево, но это его не портит. Интерстеллар, в отличие от двух остальных - нормальная такая НФ в стиле семидесятых. Ты бы еще на Одиссею прогнал, мамкин мезантрап. И даже был бы прав, но все равно красиво и заебато, а твердоебы пусть нахуй идут.

>>262655
Да. Там цирк полный.
Аноним 15/02/16 Пнд 22:05:41 #228 №262659 
>>262655
Лол, ты смотрел гроветацию? Там орбиты одинаковых высот различаются в скорости, как его можно было назвать научной фантастикой, а не фэнтези? Даже автар тверже.
Аноним 15/02/16 Пнд 22:09:10 #229 №262662 
>>262658
Интерстеллар может сойти за норм лишь на фоне другого говна, которое сейчас гордо и необоснованно называют научной фантастикой, но если одно говно является меньшим говном, чем другое говно, то оно все равно не перестает быть говном. В придачу к этому идет тележка ноланофанбоев, которые бы назвали шедевром вообще любое говно, главное лишь бы его высрал их кумир.
Аноним 15/02/16 Пнд 22:09:11 #230 №262663 
>>262658
Двачую, там драма, а черные дыры и звездолеты только для фона, который почти незаметен.
-->mov
Аноним 15/02/16 Пнд 22:09:11 #231 №262664 
>>262659
>орбиты одинаковых высот различаются в скорости
Разные орбитальные элементы? Круговая и эллиптическая, например.
Аноним 15/02/16 Пнд 22:20:20 #232 №262670 
>>262659
Это просто 3D. И нихуя больше.
Аноним 15/02/16 Пнд 22:20:34 #233 №262671 
>>262664
Тогда как мусор догоняет и обгоняет каждый спутник на круговой орбите?
Аноним 15/02/16 Пнд 22:21:59 #234 №262672 
>>262664
Там локация. В одном углу МКС, в другом Хаббл. Китайская станция посередине болтается. И ОПЯТЬ эти русские взорвали что-то и нам пиздец.
Аноним 15/02/16 Пнд 22:25:16 #235 №262674 
>>262657
Cука, из-за тебя жопу разворотило. Нахуя снимать псевдонаучные фильмы?!
Аноним 15/02/16 Пнд 22:26:20 #236 №262675 
Котаны, а верно, что внутри любого планетоида - горячо? Т.е. даже внутри какого-нибудь темного и ледяного Харона внутри килотонны геотермальной энергии и тысячеградусное раскаленное ядро?
Аноним 15/02/16 Пнд 22:40:51 #237 №262677 
Знач так, берем ракету (а лучше 5), загружаем в пару из них бомб (да помощнее) и хуярим их об ледяные шапки марса, потом туда засылаем корабль со всякими живыми бактериями, мхом, водорослями, ленточными червями, хохлами и тд. Вопрос, прокатит чтобы хотя бы появились микроорганизмы?
Аноним 15/02/16 Пнд 22:47:51 #238 №262679 
>>262677
Прокатит и без бомбардировок. Разные бактерии и микроорганизмы с Земли могут выжить на Марсе. Но зачем это делать?

Мы наоборот хотим сохранить Марс чистым и не заражать его нашей жизнью, чтобы наблюдать его в первозданной форме. Если там есть местная жизнь, то мы хотим её изучить и не хотим запускать конкурентов с нашей планеты, которые создадут путаницу.

Тем более бомбить шапки никто не будет, пока мы их хотя бы 200 лет не изучим. Или даже дольше. Никакой спешки нет. Чтобы что-то изменить - нужно это изучить. Нужно изучать почвы Марса, его климат, погоду, химию Марса, круговороты веществ. Только откровенный долбаеб предложит скинуть бомбы на шапки Марса - не зная что произойдет после этого, но надеясь на лучшее.
Аноним 15/02/16 Пнд 23:09:11 #239 №262682 
>>262679
А как же магнитное поле? Можно ебнуть по коре чтоб она притормозила, а ядро таким образом закрутилось быстрее. Магнитное поле это ведь круто для жизни.
Аноним 15/02/16 Пнд 23:19:12 #240 №262688 
>>262682
Чтобы узнать возможно ли это в принципе, нужно продвинуться очень далеко в понимании глубинной геологии Марса. Для этого нужно изучать кору, мантию и ядро Марса на протяжении многих десятилетий, если не столетий.

Или же ты опять предлагаешь поставить эксперимент планетарного масштаба не зная основ, на которые этот эксперимент повлияет? Типа у нас есть поциент и он не дышит, а давайте-ка мы в него выстрелим из дробовика - вдруг это его вылечит?

Такова рода эксперименты могут иметь очень непредсказуемые, далекоидущие и непоправимые последствия. И вообще можно добиться обратного эффекта тому, на который ты изначально рассчитывал.
Аноним 15/02/16 Пнд 23:19:15 #241 №262689 
>>262677
Зачем?
Аноним 15/02/16 Пнд 23:19:24 #242 №262690 
>>262677
>об ледяные шапки марса
Пук и пшик.
>со всякими живыми бактериями, мхом, водорослями, ленточными червями,
Которые там замерзнут на хуй, а к земе их еще засыпет метром-другим мороженной углекислоты.
Аноним 15/02/16 Пнд 23:20:29 #243 №262692 
>>262690
>к зиме
починил
Аноним 15/02/16 Пнд 23:23:29 #244 №262695 
>>262682
Магнитное поле не всегда круто для жизни. У Юпитера магнитное поле убивает.
Аноним 15/02/16 Пнд 23:23:40 #245 №262696 
>>262677
Намного эффективнее переместить Цереру к Марсу.
Аноним 15/02/16 Пнд 23:24:24 #246 №262697 
>>262696
Намного эффективнее переместить твою мамку к Марсу.
Аноним 15/02/16 Пнд 23:25:12 #247 №262698 
>>262695
Сравнивает магнитное поле камня 2х2 и газового гиганта
Аноним 15/02/16 Пнд 23:26:34 #248 №262701 
>>262696
Если бы я умел двигать планеты, то я бы поступил еще проще и сдвинул Марс ближе к Солнцу и не заморачивался бы.
Аноним 15/02/16 Пнд 23:27:05 #249 №262702 
>>262697
Она умерла, клоун. Вот твоя в самый раз.
Аноним 15/02/16 Пнд 23:27:27 #250 №262703 
>>262695
>убивает
Нихуя.
Аноним 15/02/16 Пнд 23:28:35 #251 №262706 
>>262698
Где я его сравнивал, даун?

>>262703
Капчуешь с Европы с 43 минутной задержкой?
Аноним 15/02/16 Пнд 23:29:43 #252 №262707 
>>262695
Это радиационное поле убивает
Аноним 15/02/16 Пнд 23:30:48 #253 №262708 
>>262701
А сранная куда тогда? С солнечным ветром что будешь делать? Тебе надо разогреть ядро и вернуть геологическую активность.
Аноним 15/02/16 Пнд 23:30:57 #254 №262709 
>>262707
А оно там по каким линиям распределено? Не по магнитным ли?
Аноним 15/02/16 Пнд 23:32:22 #255 №262711 
>>262708
Если я умею двигать планеты, то я могу и оживить ядро даже без спутников - очевидно же. Заебашу гигантскую термоядерную боеголовку прям в центр планеты через шахту. Авось раскочегарится.
Аноним 15/02/16 Пнд 23:32:52 #256 №262712 
>>262709
Нет.
Так какова смертельная напряженность магнитного поля?
Аноним 15/02/16 Пнд 23:34:51 #257 №262713 
>>262712
Без магнитного поля заряженное говно от Ио не будет распространяться за пределы его мелкой ссаной орбитки, так что я виню именно магнитное поле.
Аноним 15/02/16 Пнд 23:35:22 #258 №262714 DELETED
1
Аноним 15/02/16 Пнд 23:38:09 #259 №262715 
>>262711
Через какое то время оно все равно застынет.
Аноним 15/02/16 Пнд 23:42:54 #260 №262720 
>>262715
Слушай, в рамках этой гипотетической ситуации - я наделен гигантскими силами и энергиями достаточными для изменения планетарных орбит. Не шути со мной, сучечка. Если я сказал, что у меня ядро будет гореть, то оно будет гореть нахуй, даже если для этого мне придется выделить из марсианского и астероидного грунта тысячи миллиардов тонн урана и доставить эти миллиарды урановых пилюль прямиком к марсианскому ядру.
Аноним 16/02/16 Втр 00:14:52 #261 №262728 
14555708924880.jpg
>>262720
Аноним 16/02/16 Втр 08:09:38 #262 №262763 
1) По какому принципу работают крутящиеся орбитальные станции как в одиссее например? Какое притяжение будет на оси вращения? Какое будет посередине?
2) Почему в интерстелларе выводили корабль на орбиту 5 Сатурном? Откуда он у них и почему не нашлось ничего поновее?
3) Почему электронику на космических кораблях не заливают аналогом эпоксидки, которая бы защищал ее от радиации и космической пыли?
4) Зачем нужны космонавты на мкс? Чем они там заняты?
Аноним 16/02/16 Втр 08:14:27 #263 №262764 
>>262763
"Почему ветер дует" забыл.
Аноним 16/02/16 Втр 08:17:43 #264 №262765 
>>262764
Нет, не забыл
Хули ты доебался, это тупых вопросов тред, захочу и про ветер спрошу, умник блядь.
Аноним 16/02/16 Втр 08:30:29 #265 №262768 
>>262763
1. Кури учебник физики за 7й класс. Центробежная сила.
2. Пушто продавцы ностальгии продают ее патриотичным реднекам.
3. Эпоксидка не защищает от радиации.
4. Работой.
Аноним 16/02/16 Втр 08:43:15 #266 №262769 
>>262763
>Чем они там заняты?
http://www.mcc.rsa.ru/exp.htm
Аноним 16/02/16 Втр 08:56:36 #267 №262770 
>>262769
Это несет реальную пользу или космические агентства вынуждены хоть какой то смысл в полетах искать, чтобы их финансирования не лишали?
Аноним 16/02/16 Втр 09:03:00 #268 №262771 
>>262770
совершенствование материалв и технологий для космических аппаратов, исследование земли
Это летающая космическая лаборатория
>>262763
4. лаборанты и эникейщики по станции и подопытные за одно.
короче восьмирукий семихуй.
Аноним 16/02/16 Втр 10:15:34 #269 №262774 
14556069350580.jpg
Типа пиклейт
Две чд летят "навстречу" друг другу, по такой тракетории, что горизонты событий при столкновении слегка касаются
Пусть они летят очень быстро, в 1/2 скорости света
При этом рядом тусуется пожираемая обоими звезда (ну или пусть за каждой летит своя персональная еда)

Что произойдет, когда горизонты столкнуться?
Чд сразуже начнут вращаться друг вокруг друга, не размыкая горизонт событий, либо все же пролетят мимо/начнут кружиться, тоесть горизонты разорвуться? если это так то неужели никакое одинокий фотон или частича не вырвется наружу из чд в этом месте? Его ведь раздирают одинакого во все стороны, получается мы будем иметь частицу, которая "видела" что внутри чд?
Или все, что оттуда удерет в любом случе юудет не больше излучения хоккинга по размеру?

на каком этапе рассуждеий я обосрался?
Аноним 16/02/16 Втр 11:22:21 #270 №262777 
>>262774
Не иксперт, но начнём с того, что система отсчёта НЕПОНЯТНА. Да и чёрные дыры будут вращаться вокруг центра масс, а не лететь друг к другу.
Аноним 16/02/16 Втр 13:13:31 #271 №262781 
>>262774
>При этом рядом тусуется пожираемая обоими звезда (ну или пусть за каждой летит своя персональная еда)
Будут выпизданы из системы под жопу, с невероятной скоростью
Аноним 16/02/16 Втр 13:50:08 #272 №262787 
>>261274
>когда запланирована первая экспедиция на красную планету, сколько человек в ней участвует и сколько она должна длиться по времени
1)Сразу после постройки лунной базы для добычи гелия
2)Маск планирует миллион послать, вроде
3)ну, в неделю уложимся
Аноним 16/02/16 Втр 13:52:14 #273 №262789 
>>262787
>для добычи гелия
А вот зачем он нужен в промышленных масштабах?
Аноним 16/02/16 Втр 13:52:26 #274 №262790 
>>262653
В том то и дело, что в интерстелларе куча нефальсифицируемых на данный момент допущений. В Марсианине вся наука вполне подвержена критическому взгляду, поэтому съехать на "неподтвержденные" теории не выйдет.
>>262648
>Это бесконечное топливо когда корабль по два раза туда-сюда летает без дозаправки
Насколько я помню, было решено потратить топливо, отписанное на маневры для хорошего входа в атмосферу. Потратить на дополнительный импульс. И прочее. Собственно непонятно, почему ты считаешь его бесконечным, если на равномерное прямолинейное движение влияла только гравитация Земли и Марса.
>этот прокол перчатки
Ну идея в принципе рабочая, только вот успешно управлять ею так он бы не смог. Закрутило бы однозначно.
>картоха в мертвой марсианской почве
Тут надо у ботаников уточнить, но вроде как не с потолка эта идея взята.
>блядский брезент вместе обтекателя
Тут да, скорее всего перегнули в угоду зрелищу. Однако марсианская атмосфера все таки оче разреженна.


Аноним 16/02/16 Втр 13:54:32 #275 №262796 
>>262789
Нахуй не нужен.
Аноним 16/02/16 Втр 13:54:40 #276 №262797 
>>262790
> если на равномерное прямолинейное движение влияла только гравитация Земли и Марса
Но ведь для этого нужна выгодная конфигурация. Инус на суперкомпе подкрутил как надо? Или просто оче повезло всем этим лаки бастардам?
Аноним 16/02/16 Втр 14:16:07 #277 №262804 
>>262789
Шарики надувать.
Аноним 16/02/16 Втр 14:56:05 #278 №262808 
>>262804
Еще можно делать смешной голос
Аноним 16/02/16 Втр 15:10:27 #279 №262811 
Но ведь на луне можно копать гелий-3. Он нужен зачем-то? Может термоядерный реактор топить?
Аноним 16/02/16 Втр 15:28:31 #280 №262814 
Этот>>262671 прав. Почему-то в обсуждении граицапы этот момент оче редко всплывал, хотя на мой взгляд это самый главный косяк. Мусор каждые 90 минут прилетает назад к булке. 90 минут - стандартная скорость вращения по орбите МКС. Встречной скорости в 8 км/с на низкой орбите Земли в классической механике никак не реализовать кароч.
Аноним 16/02/16 Втр 16:23:01 #281 №262820 
>>262811
Много ты их видишь вокруг? Нечего пока топить.
И даже если было бы чего, гелием-3 топить это сильно отвязанная от реалий настоящео фантастика.
Аноним 16/02/16 Втр 16:37:11 #282 №262825 
>>262811
Если так приспичило жечь гелий-3 в реакторе, просто жги чистый дейтерий и половина реакций будет выдавать гелий-3, не надо никуда лететь. Реактор только сначала запили.
Аноним 16/02/16 Втр 16:39:12 #283 №262827 
>>262820
>>262825
Че пристали? Нет у меня реактора. Я просто хочу знать какой профит можно поиметь с Луны.
Аноним 16/02/16 Втр 16:43:16 #284 №262829 
>>262827
>Я просто хочу знать какой профит можно поиметь с Луны.
Да никакого, флаг там уже воткнули.
Аноним 16/02/16 Втр 16:45:18 #285 №262830 
>>262827
Профит? Никакой, только исследования, исследовать там есть чего.

Теоретически можно воду с полярных шапок добывать и разлагать на горючку до марса. Вон насо исследования гоняет на эту тему.
Аноним 16/02/16 Втр 16:47:49 #286 №262831 
>>262830
На Луне есть вода в не связанном виде?
Аноним 16/02/16 Втр 17:04:30 #287 №262834 
>>262831
Лед это несвязанная форма?
Аноним 16/02/16 Втр 17:16:22 #288 №262840 
>>262834
Таки да. Связанная это гидраты всякие.
Если я конечно еще не все учебники со школы прокурил и пропил.
Аноним 16/02/16 Втр 17:28:04 #289 №262845 
>>262840
Значит есть
Аноним 16/02/16 Втр 21:05:38 #290 №263018 
>>262774
бамп вопросу
http://www.youtube.com/watch?time_continue=12&v=ENd8Sz0AFOk
Аноним 16/02/16 Втр 21:30:52 #291 №263027 
>>263018
Хуле бамп-то?
Про звезды тебе ответили. Звезда\звезды будут отправлены нахуй сразу. В системе трех и более тел у наименее массивных судьба одна: "NАHUI POOTESHESTVUY" называется.
>Что произойдет, когда горизонты столкнуться?
Ничего не произойдет. Горизонт - это область пространства, там нет твердой поверхности.
>горизонты разорвуться?
Сольются
>неужели никакое одинокий фотон или частича не вырвется наружу из чд в этом месте?
Нет.
Аноним 16/02/16 Втр 21:38:49 #292 №263028 
14556479291270.jpg
>>263018
Если горизонты сомкнутся, то их уже ничто не "разорвет". В остальном: выбросе массы, конфигурации нового горизонта, грав.волнах, прочем - тебе никто не скажет. Ни в этом треде, ни в насе. Это пока слишком сложные и индивидуальные модели.

Вот, анон шеб-цуиньки постил интересные. >>260761
Аноним 16/02/16 Втр 21:40:44 #293 №263030 
>>263027
>В системе трех и более тел у наименее массивных судьба одна: "NАHUI POOTESHESTVUY" называется.
Если дыры звездных масс - то всё может быть.
Аноним 16/02/16 Втр 21:43:42 #294 №263033 
>>261671
Саган, Хокинг же
Аноним 16/02/16 Втр 21:53:29 #295 №263035 
>>263030
>то всё может быть.
Иди нахуй, а. Аллах там законы физики правит кочергой на месте?
Аноним 16/02/16 Втр 22:08:13 #296 №263043 
>>263035
Ой, ну да - если ЧД, то "пиздец-пиздец". Если масса дыры и звезды сопоставимы, то вокруг чего они будут вращаться?
Аноним 16/02/16 Втр 22:09:11 #297 №263045 
>>263027
>В системе трех и более тел у наименее массивных судьба одна: "NАHUI POOTESHESTVUY" называется.
Как же шаровые скопления существуют тогда? И ведь стабильные орбиты, относительно.
Аноним 16/02/16 Втр 22:20:51 #298 №263053 
>>263045
Может дело в том, что звезды в скоплении из одного газового облака постоянно рождаются и после этого им уже нет причин разлетаться?
Аноним 16/02/16 Втр 22:21:41 #299 №263055 
>>263045
Так и существуют, представь себе. С нестабильными орбитами иногда. И обмен звездами в двойных-тройных-и.т.д. системах там обыденное дело. В целом-то система стабильна, потому как системы, в которых более двух объектов иерархические.
>>263043
>если ЧД, то "пиздец-пиздец"
Да, только твои кривляния не уместны. Они пиздец не потому что страшные обосраться можно, а потому что их масса заключена в компактный объем, а массивные звезды - имеют довольно жирный радиус, и пара ЧД порвет ее как какашку.
>то вокруг чего они будут вращаться?
Вокруг общего барицентра, вокруг чего же еще.
Проблема в том, что в устойчивую (судя по пейнт-шедевру еще и тесную) систему ЧД-звезда влетает массивный компактный объект. И тут два пути: либо образуется иерархическая система, либо кто-то должен съебать из нее в закат. Учитывая то, что у нас ЧД проносятся задевая друг-друга горизонтами, я на победу звезды я бы ставить не стал. ЧД ее порвут, а остаток выпнут на мороз.
Аноним 16/02/16 Втр 22:43:24 #300 №263072 
>>262675
Что, прям никто не знает?
Аноним 16/02/16 Втр 22:52:20 #301 №263076 
14556523403930.jpg
>>263055
То, что зависимость силы гравитации от расстояния (от центра масс) растет нелинейно, еще не повод кричать "все, кто не ЧД - на мороз!". Рассчитай, при каких условиях звезды "выпнут на мороз". В самом простом приближении - ньютоновской механике, в Законах Кеплера. И будет ли иметь значение отличие размера звезды от размера ЧД.

Тут давно этот вопрос волнует ньюфагов ("Что, если Солнце вдруг превратится в черную дыру?"), а расчеты тривиальные.
Аноним 16/02/16 Втр 22:54:06 #302 №263077 
>>263072
Горячо там, горячо, давление никто не отменял. Если объект достаточно массивный, чтобы принять округлую форму, то в центре обязательно горячо.
Аноним 16/02/16 Втр 23:16:06 #303 №263093 
>>263077
>Горячо там, горячо, давление никто не отменял. Если объект достаточно массивный, чтобы принять округлую форму, то в центре обязательно горячо.
Разве только не в том случае, когда производит энергии больше, чем получает от Солнца (или близких тел)?
Аноним 16/02/16 Втр 23:44:50 #304 №263100 
14556554903920.jpg
Год на Марсе длится 687 дней, то есть гипотетически, если отправить туда смертников, они будут в 2 раза моложе чем земляне? У них замедлится старость? Спейс-уебан, но могущий в космос, требует ответа
Аноним 16/02/16 Втр 23:48:20 #305 №263104 
14556557005350.jpg
14556557005381.jpg
>>261306
>Любой проект, который потребует больше пяти лет, никогда не завершится

Это ж блять с каких хуев такое видано было?
Аноним 16/02/16 Втр 23:48:50 #306 №263106 
>>263093
Количество энергии получаемой от Солнца может варьировать с расстоянием от Солнца, а раскаленность недр зависит совсем от другого.
Аноним 17/02/16 Срд 00:00:08 #307 №263112 
>>263100
А на Венере день больше года длится, они там бессмертными станут
Аноним 17/02/16 Срд 00:00:53 #308 №263113 
>>263106
>Количество энергии получаемой от Солнца может варьировать с расстоянием от Солнца, а раскаленность недр зависит совсем от другого.
Разумеется, но второй закон термодинамики от этого не нарушается. Даже если ее недра накалены, мы это можем узнать лишь по косвенным признакам: положительна ли ее теплоотдача (с учетом разогрева, например, приливными силами близких тел). Градусник ты даже в ядро Земли не сунешь.
Аноним 17/02/16 Срд 00:05:39 #309 №263114 
>>263113
Можно геофизическими методами определить степень вязкости внутренних слоев, а следовательно и их температуру. Например сейсмологические приемники на поверхности разместить и при столкновении с небольшим астероидом анализировать скорости прохождения сейсмических волн.
Аноним 17/02/16 Срд 00:12:38 #310 №263116 
>>263112
лол, короче никак не отразится это на них, да? Плюс от жестких условий подохнуть может раньше даже
Аноним 17/02/16 Срд 00:26:43 #311 №263125 
>>263114
Только на поверхности. На какой-нибудь Седне и это будет проблематично.
Аноним 17/02/16 Срд 00:31:25 #312 №263130 
>>263116
Такое чувство что со своим шестилетним сыном разговариваю. Ебать ты аутист.
Аноним 17/02/16 Срд 00:33:34 #313 №263134 
>>263116
Да, смена освещенности, конечно, повлияет на организм, но это одно из последних, о чем стоится заботиться. В теплой куртке (как тут один товарищ предлагал) на Титане быстро ласты склеишь.
Аноним 17/02/16 Срд 00:36:15 #314 №263136 
>>263076
Рассчитывать это одними законами Кеплера - это все равно что вычислять объем куба формулой для сферы.
И отличие не просто в размерах а еще в плотности этой звезды, если система настолько компактна, что звезду раздевает ЧД, то проскакивая мимо второй такой же, на том расстоянии, что описывает тот чувак со схемой, она просто потеряет гидростатическое равновесие. Ее раздербанит приливными силами. Нахуй мне тут сидеть и считать сферических коней в вакууме из абсолютно черного газобетона, какой в этом смысл?
Аноним 17/02/16 Срд 00:43:47 #315 №263143 
14556590272640.jpg
>>263100
Год на Нептуне равен 164 годам на Земле. Пиздюк на Нептуне родившийся достигнет десятилетнего возраста через 1640 лет?
Аноним 17/02/16 Срд 00:54:58 #316 №263155 
>>263136
>Нахуй мне тут сидеть и считать сферических коней в вакууме из абсолютно черного газобетона, какой в этом смысл?
Для дыр звездных масс эффекты искажения пространства времени будут наблюдаться лишь очень близко к горизонту. Для остального - сойдет обычная орбитальная механика. Значит ты слился. Теперь брызгай слюной.
Аноним 17/02/16 Срд 01:53:41 #317 №263174 
>>263155
Ты какой-то дурачок, не могущий в удержание нити.
Перечитай пост того, кто спрашивал, потом рот разевай >>262774
Аноним 17/02/16 Срд 02:12:40 #318 №263182 
>>262774
>Что произойдет, когда горизонты столкнуться?
зачем эти фантазии, мы же знаем что произойдет, они сделают вупвуп, излучат в грав волну кучу энергии и образуют одну черную зезду смерти и продолжать сосать этого красного малыша.
Аноним 17/02/16 Срд 03:41:38 #319 №263201 
>>263174
Засчитан.
Аноним 17/02/16 Срд 03:51:45 #320 №263203 
>>263182
Если дисков не будет, и системы будут одиночными.
Аноним 17/02/16 Срд 09:25:16 #321 №263216 
>>263018
Лол в конце скатили все к биопроблемам, но не поиблися в итоге.
Аноним 17/02/16 Срд 11:29:06 #322 №263224 
14556977463280.png
Представим такую ситуацию. Союз полетел по длинному пути. В экипаже няшная тян, а командир спит непробудным сном. Камера запалит как ты няшишся с ней или есть шанс, что твой брак не развалится через пол года при возвращении?
Аноним 17/02/16 Срд 11:47:33 #323 №263231 
>>263224
В бытовом отсеке нет камеры.
Аноним 17/02/16 Срд 12:42:37 #324 №263241 
>>263130
Лол, тоже так подумал.
Аноним 17/02/16 Срд 15:26:08 #325 №263257 
>>263231
А звук?
Аноним 17/02/16 Срд 15:26:47 #326 №263258 
Если на месте Солнце была звезда другого класса, то как она выглядела с точки зрения наблюдателя? Каким был бы свет звезды? У красного карлика - алым и тусклым?
Аноним 17/02/16 Срд 15:47:32 #327 №263262 
Как Млечный пусть и Андромеда могут столкнуться, если вселенная расширяется и все галактики отдаляются друг от друга с постоянной скоростью?
Аноним 17/02/16 Срд 16:32:14 #328 №263272 
14557159343810.jpg
>>263262
Они не просто столкнутся, а скорее "сольются" в одну более крупную галактику.
Что касается противоречия, то нужно понимать следующее: действительно, далекие галактики удаляются, причем удаляются со все возрастающей скоростью. Будет ли это продолжаться вечно или со временем общая масса наблюдаемой и ненаблюдаемой материи (темной энергии и темной материи) замедлит этот процесс или даже обратит его вспять является открытым вопросом.
Что касается ближних галактик, то расширение вселенной никак не отменяет действия силы гравитации. Просто для далеких галактик эта сила очевидно ничтожна мала, а вот для тех что поближе есть варианты. В случае с Андромедой гравитация побеждает.
Впрочем, это не спасет от того что если вселенная действительно будет расширяться вечно то скорость её расширения со временем превысит световую и тогда любое слияние потеряет всякий смысл.

>>263258
Все так, и кроме того, наблюдаемый цвет звезды будет зависеть не только от её спектрального класса, но и от состава атмосферы.
Например на Марсе можно наблюдать голубоватый оттенок заката.
Если же говорить о красных карликах, то основной диапазон их излучения смещается даже не на красный, а на инфракрасный диапазон (что не способствует кстати возникновению жизни в известным нам формах)

P.S. первый раз пишу в разделе, раздел отличный, надо почаще здесь бывать.
Аноним 17/02/16 Срд 16:58:35 #329 №263275 
>>263257
Ебитесь тихо.
Аноним 17/02/16 Срд 17:40:39 #330 №263295 
>>263262
>Как Млечный пусть и Андромеда могут столкнуться, если вселенная расширяется и все галактики отдаляются друг от друга с постоянной скоростью?
Расширение "отнимает" от скорости сближения сколько-то (70 км/с, штоле), но пока не может "победить" притяжение. Так и плетемся потихоньку.
Аноним 17/02/16 Срд 19:55:14 #331 №263328 
>>263272
Даже не подумал про гравитацию, когда неделю ходил с этим ебнутым вопросом в голове. Спасибо огромное
Аноним 17/02/16 Срд 22:03:12 #332 №263338 
Наверно уже тысячу раз обсосали этот вопрос, но спрошу все равно: какой будет эффект от сбрасывания с орбиты ломика из тугоплавкого металла массой, к примеру, 50 килограмм. Каково будет поражающее действие на поверхности? А если при сбросе придавать лому дополнительное ускорение?

Также интересно, можно ли сбивать спутники, запуская по их орбитам разогнанный космический мусор, как в х/ф Гравитация?
Аноним 17/02/16 Срд 22:14:35 #333 №263341 
>>263275
Может можно выключить?
Аноним 17/02/16 Срд 22:14:59 #334 №263342 
>>263338
Пока тебя не обоссали, слушай.
Ломик никуда не упадет, а будет летать рядом и в итоге попячит сбросившего в жопу или нет.
В гравитации показали хуйню и это всем известно. Скажу тебе по сикрету. Скорость определяет орбиту, а не наоборот. За остальным иди скорее к Левантовскому. А то сейчас начнется...
Аноним 17/02/16 Срд 22:16:15 #335 №263344 
>>263341
Биомусор с биопроблемами не излечим. Пусть тянка заткнет дырку микрофона юзаным тампоном.
Аноним 17/02/16 Срд 22:38:10 #336 №263349 
>>263338
Если разумеешь английский, почитай тут: https://en.wikipedia.org/wiki/Kinetic_bombardment

А если нет, то вкратце: да, умные вояки во времена Холодной войны подумывали о том, чтобы запулить на орбиту вязанку вольфрамовых ломов и затем прицельно сводить их с орбиты на головы противника. Плюс идеи в том, что у такого дрына будет очень высокая кинетическая энергия и он будет распидорашивать все, к чему прикоснется, даже без ВВ и прочих глупостей. К тому же, пуск такого оружия очень тяжело обнаружить и защититься против него тоже практически невозможно. Минус же идеи в том, что такую палку очень тяжело точно навести на цель хотя с современным развитием точного оружия, наверное, проблема решаема, да и стоимость размещения на орбите такого оружия неоправданно высока.

Во времена войны во Вьетнаме мурриканцы пробовали использовать лайт-версию такого оружия: брали стальные болванки, приваривали к ним оперение и сбрасывали россыпью с самолетов. Эффект был схож с вертикально направленным пулеметным огнем. Земля в месте бомбардировки была словно истыкана огромной вилкой, а тела погибших пронизаны снарядами практически насквозь, от головы до живота. Такое-то орочье ОМП.

И в заключение, небезызвестный Скотт Мэнли пытался сделать что-то подобное в Kerbal Space Program. Бомбардировал титановыми ломами здание вертикальной сборки. Ничего у него не получилось: из-за особенностей симуляции дрын пролетал сквозь здание, не причиняя ему вреда.
Аноним 17/02/16 Срд 22:45:22 #337 №263350 
>>263338
Если я правильно посчитал, то сферический лом в вакууме, массой в 50 кг, разогнанный до 11 км\сек, при ударе в абсолютно твердое тело разродится взрывом 700 кг тротилового эквивалента.
>>263338
Можно запускать мусор по ретроградной орбите по отношению к движению спутника. Можно, но нахуй никто не будет заморачиваться. Дорога/нинужно.
Аноним 17/02/16 Срд 23:03:12 #338 №263353 
>>263349
>Во времена войны во Вьетнаме мурриканцы пробовали использовать лайт-версию такого оружия: брали стальные болванки, приваривали к ним оперение и сбрасывали россыпью с самолетов.
У тебя не найдётся ссылочки на это? Хочу почитать.
Аноним 17/02/16 Срд 23:06:22 #339 №263354 
>>263353
https://en.wikipedia.org/wiki/Lazy_Dog_%28bomb%29
Аноним 17/02/16 Срд 23:14:55 #340 №263355 
>>263354
Благодарю.
Аноним 18/02/16 Чтв 00:22:07 #341 №263368 
>>260135
если 3 то 3 луча?
Аноним 18/02/16 Чтв 00:25:11 #342 №263370 
>>263368
да
Аноним 18/02/16 Чтв 05:13:34 #343 №263444 
Если бы на Марсе была возможна жизнь то лететь туда все равно было бы нельзя ближайшие лет 200?

То есть прилетел, снял скафандр, ходишь, дышишь. А дальше что? Ты ведь не сможешь улететь обратно домой. Ведь атмосфера и вот это все.
Молодым колонизаторам придется из говна и палок собирать космодром и ракеты, которые могли бы стартовать с поверхности Марса в направление земли.

Или как это всё реализовывалось бы, если бы Марс был ну неким подобием Земли?
Аноним 18/02/16 Чтв 05:43:08 #344 №263447 
>>263444
также как и здесь Mars SpaceX & Co
Аноним 18/02/16 Чтв 11:11:44 #345 №263449 
>>263444
На марсе все же сила тяжести одна треть от земной.
Аноним 18/02/16 Чтв 11:23:11 #346 №263450 
>>263444
Ну, это бы сильно упростило логистику и снизило бы требования по весу. Тебе не надо копать землянки (хотя про радиацию ты не сказал), не надо тащить всякие конвертеры воздуха и говна.
Кроме того, сразу появляется несложная и интересная задача для миссии — посадить разные растения, через год-пять вернуться и посмотреть состояние огородика.
Марсван сразу был бы более адекватным проектом, который по сути бы не отличался от колонизации какой-нибудь Австралии. За счёт экономии на лайфсуппорте закидываешь ещё больше колонистов, им и попроще будет и веселее.
Аноним 18/02/16 Чтв 14:28:28 #347 №263474 
>>263444
>>263450
>>263449
>>263447
Марс не нужен. Эта ебанная гравитационная яма. Нам нужны астероиды. С их помошью можно реально все. Не можешь сдвинуть планету? - не беда! Астероиды пройдут по специальной траектории и заберут либо отдадут часть энергии для маневра. Нужна большая орбитальная станция? Астероид! Корабль? Астероид! Вода? Астероид! Гравитация? Закрути астероид вокруг оси!
Аноним 18/02/16 Чтв 15:07:29 #348 №263477 
>>263474
Жить длительное время ты как собираешься на астероиде, лол?
Аноним 18/02/16 Чтв 15:25:44 #349 №263479 
>>263477
А ты на параше без нихуя? Желаю удачи. Астероид лучше. Его легче сдвинуть ближе к солнцу.
Аноним 18/02/16 Чтв 15:31:21 #350 №263480 
>>263477
Каковом к верху ебана. На марсе хуй нормально приземлишься. Хуй выживешь, воду хуй достанешь, герметичность хуй наладишь. А самое главное он ДАЛЕКООО. Вот другой разговор астеройд эрос - Большой летает как маршрутный автобус. Прицепился углубился в грунт либо нашел пещеру и залепил пленкой в глубине.
Аноним 18/02/16 Чтв 15:31:40 #351 №263481 
1. Можно ли использовать недавно найденные гравитационные волны для разгона межзвёздных зондов? Чисто в теории. Построить волшебный ускоритель, который сфокусирует волны и направить, куда нужно. Нет, скорее всего. Вряд ли их энергия достаточно велика.
2. Так выяснили, что изменило курс "Пионера"?
Аноним 18/02/16 Чтв 15:36:25 #352 №263482 
>>263481
Волны от черных дыр практически использовать - только в качестве астронавигации для прыжка.
А вот для самого прыжка нужен ПУЗЫРЬ МЕКСИКАНЦА.
Аноним 18/02/16 Чтв 15:40:15 #353 №263483 
>>263481
>Так выяснили, что изменило курс "Пионера"?
Микро-астероид.
Аноним 18/02/16 Чтв 15:46:53 #354 №263484 
>>263480
> Прицепился углубился в грунт либо нашел пещеру и залепил пленкой в глубине.
Мамка тебя и там достанет, сычуш
Аноним 18/02/16 Чтв 15:52:57 #355 №263485 
>>263480
Что это за дивный манямир у тебя?
Аноним 18/02/16 Чтв 15:57:21 #356 №263487 
>>263480
Осталось изобрести исскуственную гравитацию и можно жить
Аноним 18/02/16 Чтв 16:11:37 #357 №263493 
>>263481
>Так выяснили, что изменило курс "Пионера"?
Ну вроде как да, третья открытая проблема ОТО в Солнечной системе считается решенной:
https://ru.wikipedia.org/wiki/Эффект_«Пионера»

Неправильно рассчитали отдачу тепла через излучение. Она и дает этот малый вклад в изменение скорости.
Аноним 18/02/16 Чтв 17:37:30 #358 №263503 
>>263474
>Закрути астероид вокруг оси
И как это поможет? Или у тебя полый астероид?
Аноним 18/02/16 Чтв 17:52:03 #359 №263505 
>>263474
>Не можешь сдвинуть планету?
и не сможешь, именно так это сделать, масса астероида ничтожна.

Мантра настоящего спейсмана:
Я против планетарных колоний.
Так как они не отвечают ни одному требованию которые мы постулируем для колонии на данный момент, и так как они не решают ни одной проблемы, из тех которые мы видим на данный момент.

Я против колоний в астероидах, т.е. внутри самих астероидов. Из-за технических проблем, из-за проблем мобильности, из-за нерационального расходования ресурсов, из-за необходимости подстраиваться под астероид вместо строительства того что надо и развязывания рук в будущем, из-за серьезной технической сложности перехода в такую колонию.

Я за автономные орбитальные поселения: колонии О’Нейла, хабитаты, банки и тд.
Я за луну как сырьевую базу для постройки первых колоний, и возможно их строительства в будущем.
Аноним 18/02/16 Чтв 17:59:29 #360 №263506 
>>263505
>масса астероида ничтожна
>эрос весит 6.5 миллиардов тонн
>ничтожна


Аноним 18/02/16 Чтв 18:04:51 #361 №263507 
Почему большая часть звезд в галактике лежат на одной плоскости(плоскостью это сложно назвать конечно, но все же) а не разбросаны друг от друга так, что в целом образуют шар?
Аноним 18/02/16 Чтв 18:05:37 #362 №263509 
>>263506
>эрос весит 6.5 миллиардов тонн
А в сравнении с планетой?
Аноним 18/02/16 Чтв 18:09:55 #363 №263511 
>>263507
Экватор солнца же, ебана.
Аноним 18/02/16 Чтв 18:10:24 #364 №263512 
>>263507
Потому что галактика вращается. Центростремительное ускорение противодействует гравитации черной дыры, которая стремится уронить на себя все вещество галактики. Но центростремительное ускорение действует только в одной плоскости - плоскости вращения галактики, тогда как гравитация действует во все стороны. Таким образом, облако из звезд, кторым когда-то была наша галактика, сплющивается и растягивается в относительно плоский диск.
Тащемта та же причина, по которой планеты в солнечной (или луны в планетарной) системе движутся примерно в одной плоскости.
Или диск у Сатурна. Та же самая история.
Аноним 18/02/16 Чтв 18:11:56 #365 №263513 
>>263511
>>263512
Спасибо, думал что тут сложнее будет, а оказалось все намного проще
Аноним 18/02/16 Чтв 18:12:17 #366 №263514 
14558083379640.gif
>>263507
Плюс, существуют и эллиптические галактики. Это нам так повезло.
Аноним 18/02/16 Чтв 18:13:43 #367 №263515 
>>263514
SBa жутко выглядит, как-то слишком правильная форма для естественного объекта
Аноним 18/02/16 Чтв 18:16:12 #368 №263517 
>>263515
Хотя че я, обычное же дело, таких хреновен по космосу полно, тот же пятиугольник на северном полюсе Сатурна
Аноним 18/02/16 Чтв 18:18:06 #369 №263518 
14558086865540.jpg
>>263515
Это просто на картинке ее так изобразили. На деле они выглядят естественнее.
Аноним 18/02/16 Чтв 18:27:54 #370 №263521 
>>263509
Да хоть в сравнении с звёздой. Двигать его все равно не выйдет
Аноним 18/02/16 Чтв 18:32:31 #371 №263524 
>>263521
>Да хоть в сравнении с звёздой. Двигать его все равно не выйдет
Значит ты сравниваешь его с человеком, или чем-то близким. В любом случае, это лишь сравнение. Он не большой и не маленький.
Аноним 18/02/16 Чтв 18:37:11 #372 №263526 
>>263524
Демагог сраный
Аноним 18/02/16 Чтв 18:40:39 #373 №263528 DELETED
>>263526
Пiйшов нахуй.
Аноним 18/02/16 Чтв 18:42:01 #374 №263529 DELETED
>>263528
Норм завизжал, Тарас.
Аноним 18/02/16 Чтв 18:44:09 #375 №263530 DELETED
>>263528
Лесь Подервянский, ты? Когда моар будет?
Аноним 18/02/16 Чтв 18:44:12 #376 №263531 DELETED
>>263526
>>263528
>>263529
Хватит петушиться, долбоёбы. У нас тут приличное место.
Аноним 18/02/16 Чтв 19:20:26 #377 №263538 
>>263521
Я неправильно тебя понял. Подумал, что ты это собрался астероидами планеты двигать.

Сам астероид двигать, конечно, будет тяжело, согласен.
Аноним 18/02/16 Чтв 21:46:48 #378 №263585 
>>263538
двигать астероид, таки можно, особенно если не торопится или грамотно подойти к вопросу.
с планетой все сложнее, это как подвинуть все астероиды разом, только в 10000 раз сложнее.
Короче теоретически возможно, но геморно и нахуй не нужно.
возможно уже я не понял этого оратора >>263474
можномжно он был так окрылен возможностями что просто заговорился.
Астероиды это круто, безусловно.
Аноним 19/02/16 Птн 01:08:24 #379 №263625 
>>263482
А как же пульсары? Их легче искать, быстрее. Хотя, для тех, кто может в пузырь, это не имеет значения.
mkmm [dadaddad] mkmkm 19/02/16 Птн 01:27:50 #380 №263628 
mkmkm
Аноним 19/02/16 Птн 10:09:05 #381 №263697 
14558657458700.jpg
>>263517
Таки шестиугольник!
Аноним 19/02/16 Птн 11:02:57 #382 №263698 
>>263518
кто их раскрашивает, интересно?
Аноним 19/02/16 Птн 11:51:39 #383 №263704 
Почему Венера вращается в другую сторону? НИТАКАЯКАКФСЕ дохуя?
Аноним 19/02/16 Птн 12:03:56 #384 №263705 
14558726362660.png
>>263697
Аноним 19/02/16 Птн 12:12:27 #385 №263706 
14558731473190.jpg
>>263704
>Почему Венера вращается в другую сторону?
Поясню на всякий случай, речь идет о вращении вокруг своей оси. По орбите Венера летает в ту же сторону, как и все остальные, не выебывается.
Перевернулась. Вращалась, вращалась, как все, а потом - хоп! - перевернулась вверх тормашками и теперь вращается наоборот.

А Уран, например, завалился на бок и так теперь и летает. Поэтому он был такой скучный, когда мимо него Вояджер-2 летел. Планета тогда была обращена к солнцу одним своим полюсом, и погода на ней стала очень унылой и однообразной: на одной стороне, постоянно обращенной к свету, газы нагреваются и стремительный домкратом летят на обратную, холодную сторону, мерзнущую в вечной тьме. Никакой смены дня и ночи, никакого бурления говн, никаких красивеньких облачков, видных из космоса.
Говорят, теперь Уран таки повернулся к солнышку бочком и теперь должен выглядеть поинтереснее.
Аноним 19/02/16 Птн 12:16:22 #386 №263707 
>>263698
Сидит какой-нибудь интерн в НАСЕ и раскрашивает, наверное.
Аноним 19/02/16 Птн 12:19:33 #387 №263709 
>>263704
Мне больше интересно что бы было, если бы она вращалась по ретроградной орбите. Охуеешь тормозить у нее тогда, сложнее чем на солнце упасть.
Аноним 19/02/16 Птн 13:07:21 #388 №263712 
>>263707
>интерн в НАСЕ
Аноним 19/02/16 Птн 13:33:20 #389 №263713 
>>263712
Словно штото плохое. У них довольно обширная программа.
Аноним 19/02/16 Птн 23:49:01 #390 №263803 
​А что, если гравитации нет, а то, что мы под ней подразумеваем, на самом деле эффект электромагнетизма?
И что, если эта электромагнетическая гравитация имеет предел распространения, дальше которого сила становится ниже кванта притяжения, то есть пропадает совсем? Это бы избавило нас от тёмной материи.
Аноним 20/02/16 Суб 00:47:59 #391 №263812 
Если 2 параллельных зеркала поставить напротив друг друга и пустить луч свет между ними, то будут ли они ускоряться бесконечно. Если взять коэффициент отражения близкий к 100.
Аноним 20/02/16 Суб 01:47:59 #392 №263818 
Есть ли надежды антошки, перспективы и вообще, стеснят ли эти йобы в будущем ракеты-носители?
Аноним 20/02/16 Суб 03:30:22 #393 №263882 
>>263818
Есть вероятность. Если придумать как выбрасывать небольшую массу топлива с опиздохуительной скоростью, то может быть.
Аноним 20/02/16 Суб 05:39:16 #394 №263917 
>>263812
нет
если 100 то тоже нет
Аноним 20/02/16 Суб 07:56:51 #395 №263925 
>>263917
Если 100 - да. Если это идеальные зеркала, и фотон будет бить ровно в центр.
Аноним 20/02/16 Суб 08:08:21 #396 №263926 
>>263925
Нет.
Фотон уйдет от источника света в сторону первого зеркала, отразится и уйдет обратно в источник. До второго даже не дойдет. Это если твоим "языком" говорить.
Аноним 20/02/16 Суб 08:34:27 #397 №263928 
Посеняются ли орбиты планет, если на место Солнца поставить Чёрную дыру с той же массой? А если с тем же размером?
Аноним 20/02/16 Суб 08:36:36 #398 №263929 
>>263706
>>263709
Интересно, а существуют ли планеты в нашей вселенной, вращающиеся по ретроградной орбите вокруг своих звёзд?
Аноним 20/02/16 Суб 10:04:11 #399 №263931 
>>263929
Если планета не образовалась в звездной системе в результате аккреции, но была захвачена звездой из межзвездного пространства, она может как угодно вращаться: хоть по ретроградной орбите, хоть вообще перпендикулярно плоскости эклиптики. Но тогда ее орбита вряд ли будет круговой. Скорее, будет летать по очень вытянутой орбите, как комета.
Аноним 20/02/16 Суб 10:44:12 #400 №263937 
>>263928
нет да
Аноним 20/02/16 Суб 10:53:56 #401 №263938 
Есть ли космодромы с которых с ходу можно выводить спутники на экваториальную орбиту?
Аноним 20/02/16 Суб 10:55:33 #402 №263939 
>>263938
Куру вроде.
Аноним 20/02/16 Суб 11:15:23 #403 №263941 
>>263926
>>Фотон уйдет от источника света в сторону первого зеркала, отразится и уйдет обратно в источник. До второго даже не дойдет.
Почему обратно в источник? Ставим зеркала в космосе на огромное расстояние и пока фотоны летят до первого зеркала убираем источник света. Этим источником могла быть аннигиляция вещества и антивещества.
Аноним 20/02/16 Суб 11:46:52 #404 №263956 
Сколько нужно денег чтобы основать собственную космическую компанию для запуска мелкоспутников? Например покупать списываемые МБР и запускать с них.
Аноним 20/02/16 Суб 12:00:17 #405 №263959 
>>263956
Нужны не деньги, нужны ресурсы.
https://youtu.be/BCuadzlts4o
Аноним 20/02/16 Суб 12:31:59 #406 №263964 
>>263956
Просто покупать мбр не получится никто такое тебе не прдаст. Пуск нужно будет покупать у оператора, который проведет модификацию ракеты, сделает и приаттачит к ней РБ. В этой схеме ты, как посредник
лишний, такие услуги давно предоставляются.
Аноним 20/02/16 Суб 12:42:50 #407 №263967 
test
Аноним 20/02/16 Суб 12:52:50 #408 №263969 
>>263964
Ну ок, а есть данные какой профит от запуска конверсионной ракеты, т.е. чистая прибыль?
Аноним 20/02/16 Суб 12:53:22 #409 №263970 
>>263969
И вдогонку - интересен принцип страхования запусков.
Аноним 20/02/16 Суб 14:38:51 #410 №263984 
>>263969
>>263970
http://www.eurockot.com/faq/commercial-financial-insurance/
Аноним 20/02/16 Суб 16:20:22 #411 №264005 
>>263984
>евроскот

Сука, теперь развидеть не могу.
Аноним 20/02/16 Суб 17:03:02 #412 №264009 
>>264005
Надо это зафорсить.
Аноним 20/02/16 Суб 17:04:46 #413 №264010 
>>264009
Нет не надо.
Аноним 20/02/16 Суб 17:05:13 #414 №264011 
>>264009
Поздно форсить, Рокот списывать хотят. Неизвестно, будут ли еще полеты на нем, и сколько.
Аноним 20/02/16 Суб 17:35:52 #415 №264014 
Поясните какой шанс на успех в наше время имеет афера наподобие лунной? Ну т.е. снять в павильоне например высадку на Марс и выдать это за чистую монету.
Аноним 20/02/16 Суб 17:56:02 #416 №264017 
>>264014
Ты разделы попутал, это не /zog/
Аноним 20/02/16 Суб 18:51:27 #417 №264021 
>>263925
если 100 то нет, так как энергия фотона квантуется
и так как эта энергия переходит в кинетическую энергию пластин.

В целом то мысль правильная, но зачем это бесконечное ускорение, когда достаточно 90 процентов перехода энергии фотонов в кинетическую энергию крафта, это уже существенно выше кпд фотонного двигателя.
Аноним 20/02/16 Суб 20:29:46 #418 №264028 
Знаки зодиака вообще трушные?
Ну тип рил влияет число под которым родился человек на его характер?
Аноним 20/02/16 Суб 20:42:09 #419 №264031 
14559901300580.jpg
Почему на фотках с МКС звезд нихуя не видно (читал, якобы в космосе они не выделяют достаточно света, чтобы их было видно на снимке). Тогда почему с Хаббла звезды, планеты, туманности хорошо видны или их блять тоже дорисовывают фотожаберы с НАСА? и видят ли звезды в космосе астронавты и насколько это отличается от того, что мы видим на земле?
Аноним 20/02/16 Суб 20:48:48 #420 №264034 
>>264014
0
столько систем мониторинга запусков.
Один норад позволит любому нубу развенчать твой скам, не успеешь даже хайп создать.
трекер какойто http://www.n2yo.com/database/
анон подскажет тру трекер
Аноним 20/02/16 Суб 20:51:01 #421 №264036 
>>264028
шел бы ты отсюда, в сцы там
они не влияют, а вот сезон рождения, и папа с мамой вполне запросто.
Аноним 20/02/16 Суб 21:49:22 #422 №264053 
>>264028
Астрология - наебалово для дебилов. Нет, не влияет.
Аноним 20/02/16 Суб 21:51:35 #423 №264055 
Котаны, а за сколько времени большой бесформенный кусок камня вроде Луны становится сферой? Для чистоты вопроса (чтобы не было - а, он таким формируется), скажем, после массив импакт?
Аноним 20/02/16 Суб 22:06:43 #424 №264057 
14559952038970.jpg
14559952039001.jpg
14559952039022.jpg
14559952039033.jpg
>>264031
>Почему на фотках с МКС звезд нихуя не видно
Вообще видно, но динамический диапазон камер не позволяет. Либо пересеченное окружение и звезды, либо все темное, но звезды.
пик 1 - темный Купол и звезды. Пик 2 -светлый купол но не видно звезд.
Если подобрать правильно "время суток" и соответствующим образом выбрать режим съемки, то можно получить пик 3 и 4. Но, как видно, выкручивают ISO и появляются жуткие шумы.
Аноним 20/02/16 Суб 22:13:53 #425 №264059 
>>264055
слишком много хочешь, почитай про вязкость и во сколько она оценивается для земли луны
и посчитай за сколько оно примет форму при удоблетворяющих тебя условиях из интересующей тебя формы.
У планет есть оценочные параметры вязкости.
Аноним 20/02/16 Суб 22:18:39 #426 №264060 
>>264031
>почему с Хаббла звезды, планеты, туманности хорошо видны
Потому что Хаббл снимает объекты часами, фотографируя темные объекты и в его поле зрения больше ничего не попадает.
>или их блять тоже дорисовывают фотожаберы с НАСА?
КРУГОМ ФОТОШОП ВСЕ ВРУТ ВЛАСТИ СКРЫВАЮТ НЕТ НИКАКИХ ГАЛАКТИК ВСЕ НАРИСАВАЛИ СТАЖЕРЫ ИЗ НАСА
>и насколько это отличается от того, что мы видим на земле?
Например там звезды не мерцают.
Аноним 20/02/16 Суб 22:46:42 #427 №264062 
>>264060
> КРУГОМ ФОТОШОП ВСЕ ВРУТ ВЛАСТИ СКРЫВАЮТ НЕТ НИКАКИХ ГАЛАКТИК ВСЕ НАРИСАВАЛИ СТАЖЕРЫ ИЗ НАСА
Удваиваю
мимо_Бог
Аноним 20/02/16 Суб 22:49:32 #428 №264064 
>>264062
Тебя нет. Ты жидовская пропоганда.
Аноним 20/02/16 Суб 23:02:24 #429 №264067 
>>264060
но ведь снимки хабла подкрашивают в редакторах, неужели все выглядит так же ярко и красочно, как под кислотой?
Аноним 20/02/16 Суб 23:23:47 #430 №264076 
>>264064
Удваиваю
мимо_Ницше
Аноним 20/02/16 Суб 23:33:37 #431 №264080 
14560004174000.jpg
14560004174081.jpg
14560004174312.jpg
14560004174563.jpg
>>264067
Вот четыре фотки Андромеды. Какая снята Хабблом определишь?
Аноним 20/02/16 Суб 23:40:06 #432 №264083 
>>264080
воу воу, палегчи, я нихуя не понимат в этом, просто интересно было, как они подкрашивают фотки, как определяют, какого цвета та или иная область должна быть, но это пиздецки красиво, какие же мы блядь ничтожества
Аноним 20/02/16 Суб 23:42:11 #433 №264085 
>>264080
Все сняты Хабблом?
Аноним 21/02/16 Вск 00:16:05 #434 №264101 
>>264080
вота хабл https://www.youtube.com/watch?v=7q_MOEKHSXg
а это мыльное говно
Аноним 21/02/16 Вск 00:19:35 #435 №264105 
14560031760070.jpg
14560031760111.jpg
>>264085
Ни одна из них не снята Хабблом.
>>264083
>я нихуя не понимат в этом
Фотографируют через фильтры
Выделяют цвета под научные задачи, всегда можно под снимком прочитать что выделяют и каким цветом. Если ничего не выделяют, пишут "натуральные цвета"
На фотке 1995-го года выделили по цветам:
Зелёный цвет отмечает водород, красный — однократно ионизированную серу и синий — дважды ионизированные атомы кислорода
На фотке 2014 года:
Синий — кислород, оранжевый — сера, азот и водород — зелёный.
Аноним 21/02/16 Вск 00:24:37 #436 №264113 
>>264101
А вот рябь мелкая это все звезды или шум?
Аноним 21/02/16 Вск 00:26:18 #437 №264115 
14560035788660.jpg
14560035788701.jpg
>>264101
Но-но, Хаббл тоже начинал как самое натуральное мыльное говно.
Аноним 21/02/16 Вск 00:27:38 #438 №264116 
14560036581200.jpg
>>264113
Звезды
Аноним 21/02/16 Вск 00:28:46 #439 №264117 
>>264116
Ну этоже охуеть просто. МИЛЛИАРДЫ МИЛЛИАРДОВ
Аноним 21/02/16 Вск 00:32:23 #440 №264118 
>>264117
то что на переднем фоне звезды, это звезды нашей галактики
не забывай M31 находится на расстоянии 2.5 миллиона световых лет от нас
Аноним 21/02/16 Вск 00:36:36 #441 №264120 
14560041961890.jpg
>>264117
Аноним 21/02/16 Вск 01:37:47 #442 №264136 
14560078678160.jpg
Скажите уже наконец, реально ли НАСА пытается высрать варп-двигатель, искривляющий пространство или это утка для дибилов?
Аноним 21/02/16 Вск 02:03:50 #443 №264144 
>>264136
Тащемта мне кажется тут без фундаментальных открытий не обойтись! Вот как откроют что нибудь эдакое, ну тип поймут природу гравитационных волн или разберутся с этой сраной кучей измерений или струнами! Вот тогда я думаю и что нибудь да появится!
Аноним 21/02/16 Вск 02:13:22 #444 №264146 
На помните мне, что там за нестабильность кольца вокруг земли.
Условно тор, в середине дырки земля, почему распидорасит?
Аноним 21/02/16 Вск 02:17:23 #445 №264148 
>>264136
Утка для дибилов.
Аноним 21/02/16 Вск 03:33:01 #446 №264168 
>>264136
НАСА не чего не пытается высрать. НАСА проводит исследования, которые должны внести ясность, есть ли принци
Аноним 21/02/16 Вск 03:34:53 #447 №264169 
>>264168
Сук, ебаный смартфон. Принципиально теория Алькубьере правильная или нет. Принципиально можно создать этот ебаный пузырь, или нет
Аноним 21/02/16 Вск 04:13:31 #448 №264185 DELETED
>>264169
Ученый выебал журналиста, что непонятного?
Аноним 21/02/16 Вск 06:42:48 #449 №264204 
>>263941
Ну тогда с расстоянием энергия будет рассеиваться по веществу же.
Аноним 21/02/16 Вск 09:57:03 #450 №264216 
>>264169
Я уж подумал снайпер в треде
Аноним 21/02/16 Вск 11:50:53 #451 №264241 
Как и в фантастическом сериале “Звёздный путь”, в реальности принципы действия этого двигателя вполне возможны. Физик Гарольд Уайт взял за основу расчеты своего коллеги Мигеля Алькубьерре, который первый задумался о двигателе искривления как о сверхбыстром способе перемещения в системе общей теории относительности.

Он исправил погрешности в теории Алькубьерре и провёл расчёты, благодаря которым добился уменьшения затрачиваемой мощности с заряда эквивалентному энергии массы 300 планет Земля до энергии, количество которой равно энергии массы чуть более 700 килограмм.

Аэрокосмическое агентство NASA сейчас всерьёз занялось исследованием принципов реализации данной теории.

пидоровикия
Аноним 21/02/16 Вск 11:54:08 #452 №264243 
>>264241
Имеется ввиду энергия аннигиляции 700 кг вещества?
Аноним 21/02/16 Вск 12:14:40 #453 №264246 
Поясните нубу за расширение Вселенной.

Предположим, Вселенная бесконечна. После Большого Теракта она постоянно растет. Вопрос: куда она растет, если она бесконечная? Кто-то объяснил что просто расстояние между объектами Вселенной увеличивается, а не сама Вселенная.
Аноним 21/02/16 Вск 13:00:04 #454 №264249 
>>264246
>Предположим, Вселенная бесконечна.
Предположи, что это не так.
Аноним 21/02/16 Вск 13:06:35 #455 №264251 
>>264249
Тогда вопрос исчерпан. Но разве доказано, что она конечна?
Аноним 21/02/16 Вск 13:16:24 #456 №264254 
14560497842150.jpg
>>264251
Не бесконечна но беспредельна
Аноним 21/02/16 Вск 16:59:16 #457 №264297 
14560631562460.gif
>>264246
Представь себе, что ты живешь на поверхности шара. безумное предположение!
Понятное дело, что площадь шара, на котором ты живешь, не бесконечна. Но ты можешь сколько угодно идти по нему в любую сторону и никогда не дойдешь до конца. Твой шар будет казаться для тебя бесконечной двумерной плоскостью.

А теперь представь, что ты живешь на поверхности увеличивающегося шара раздувающегося воздушного шарика, например. По твоим ощущениям, твой шар бесконечен - так куда же он увеличивается? И где центр расширения? Куда бы ты ни пошел по его поверхности, центра расширения ты не найдешь - везде оно будет ощущаться одинаково. Любые точки на ощущаемой тобой плоскости будут удаляться друг от друга - причем чем дальше они друг от друга, тем быстрее. Центр расширения будет для твоего взора недоступен, потому что он будет находиться вне твоей двумерной системы координат.

А теперь представь, что "шар", на котором ты живешь, имеет 4 (5, 6, 11 - не знаю уж, сколько их там насчитали) измерений, но видишь ты его трехмерным. Собсно, это наша Вселенная.
Аноним 21/02/16 Вск 16:59:31 #458 №264298 DELETED
>>264254
Аноним 21/02/16 Вск 17:34:06 #459 №264303 
>>264297
Есть методологические трудности. Т.к. считается, что характерный размер Метагалактики меньше размера Вселенной, и что пространство топологически плоское, предполагать его замкнутость нельзя.

Ну и известный факт - нет ни единого достоверного наблюдения "близкого к нам" объекта, например, Галактики. А ведь, в таком случае, объекты должны наблюдаться сами из себя, хоть и с задержкой.
Аноним 21/02/16 Вск 18:25:51 #460 №264311 
>>264303
>нет ни единого достоверного наблюдения "близкого к нам" объекта, например, Галактики. А ведь, в таком случае, объекты должны наблюдаться сами из себя, хоть и с задержкой.
Шта блядь?
Аноним 21/02/16 Вск 18:39:04 #461 №264314 
>>264311
Находящийся на Римановой Сфере объект, например Галактика, излучает фотон. Когда этот фотон "обогнет" всю сферу по кратчайшей траектории, наблюдатель в галактике должен его увидеть.
Аноним 21/02/16 Вск 18:39:34 #462 №264315 
>>264311
Ну тип свет распространяется во все стороны же. И по-хорошему, если вселенная действительно замкнута на себя, он должен возвращаться туда, откуда улетел.
Аноним 21/02/16 Вск 18:50:43 #463 №264317 
>>264314
>>264315
Но ведь Вселенная пиздец какая большая и он вряд ли вернется из-за расширения пространства.
Аноним 21/02/16 Вск 18:58:19 #464 №264320 
>>264317
>Но ведь Вселенная пиздец какая большая и он вряд ли вернется из-за расширения пространства.
Поэтому никто и не говорит о Вселенной: слишком большая. С Метагалактикой-то дохуя неясного, а тут...
Аноним 21/02/16 Вск 22:01:26 #465 №264386 
У SLS будут твердотопливные бустеры, как я понимаю - они в НАСА там полностью необучаемые? Экипаж на старте так же, как в Шаттлах, возможности спастись не имеет?
Аноним 21/02/16 Вск 22:22:32 #466 №264396 
>>264386
А напомни мне процент взрывов твердотопливных бустеров за последние лет 20?
Аноним 21/02/16 Вск 22:24:24 #467 №264397 
>>264241
>c заряда эквивалентному энергии массы 300 планет Земля до энергии, количество которой равно энергии массы чуть более 700 килограмм.
Неужели при моей жизни ещё успеют собрать какой-нибудь прототип?
Аноним 21/02/16 Вск 22:32:50 #468 №264398 
>>264386
Имеет, есть же башня САС. Тоже твердотопливная, кстати - к чему эта твердофобия.
Аноним 22/02/16 Пнд 02:32:23 #469 №264453 
14560975435860.jpg
Посоветуйте годных книжек по космосу, желательно научно-популярных, чтобы тупому читателю подогреть интерес к космосу и его устройству
Аноним 22/02/16 Пнд 03:52:36 #470 №264467 
Вопросы терраформирования, вроде, относятся к /spc/.
Космач, а что если растерраформить Земляшку?
Допустим, в какой-то момент (условно будем считать, что это произошло мгновенно) на Земле исчезает биосфера как таковая - вместе с микробами, растениями и обезьянками (как вариант, не тупо исчезает, а умирает и гниёт ну или что оно там будет делать без микроорганизмов).
Как долго после этого атмосфера будет оставаться пригодной для дыхания человеком?
Или, если смягчить - исчезает всё, кроме какого островка в океане размером с Исландию. Т.е. растительность не распространяется по планете, но и не вымирает дальше этого предела - сможет ли такой островок "законсервировать" атмосферу планеты в её текущем виде?
Аноним 22/02/16 Пнд 05:09:10 #471 №264469 
>>264297
Аналогия с точкой на поверхности шара - говно.

Земля тоже шар, куда бы мы ни пошли, конца не будет, но это не значит, что мы не попадем в то же место, откуда начали путь. Земля не бесконечна.
Аноним 22/02/16 Пнд 06:54:33 #472 №264474 
>>264146
что с этим, никто не помнит ?
Аноним 22/02/16 Пнд 10:28:06 #473 №264487 
>>264467
Чё, дед, кукуруза больше не растет? Кислород без биосферы упадет до критических процентов в течение пары тысяч лет. Никакие островки законсервировать её не смогут, и вообще, - половина кислорода вырабатывается не джунглями Амазонки, а цианобактериями.
Аноним 22/02/16 Пнд 10:34:10 #474 №264488 
>>264487
>пары тысяч лет
Вообще похуй
Аноним 22/02/16 Пнд 10:59:00 #475 №264490 
>>264488
А в мире взрослых это "практически мгновенно". Да там, скорее, обезьяны не от кислородного голодания передОхнут.
Аноним 22/02/16 Пнд 12:06:23 #476 №264497 
>>264453
Карл Саган, "Космос".
Аноним 22/02/16 Пнд 12:13:39 #477 №264499 
>>264469
Я говорил о том, что сколько бы ты ни шел, ты никогда не дойдешь до края шара - потому что у шара края нет. А так да, шар не бесконечен как и Вселенная.

Аноним 22/02/16 Пнд 12:14:51 #478 №264500 
>>264386
Какая связь между твердотопливными бустерами и спасаемостью экипажа?
Аноним 22/02/16 Пнд 12:31:47 #479 №264502 
Почему в разных клетках живых организмов работает разный набор генов? Что определяет эту дифференциацию и специализацию? Почему клетки мозга отличаются от клеток кожи? Как они понимают что им надо быть нейронами или клетками кожи?
Аноним 22/02/16 Пнд 12:38:02 #480 №264504 
>>264502
Ты разделом ошибся
Аноним 22/02/16 Пнд 12:51:43 #481 №264507 
>>264500
Ну такая, что их нельзя выключить, наверное. Ну и они имеют привычку взрываться.
Аноним 22/02/16 Пнд 13:00:24 #482 №264508 
>>264507
>привычку взрываться
Чет не припомню такого
Аноним 22/02/16 Пнд 13:08:27 #483 №264510 
>>264487
Штобля? А где фитопланктон?
Аноним 22/02/16 Пнд 13:17:02 #484 №264511 
>>264510
>А где фитопланктон?
Ну давай это словечко использовать, оно моднее:
https://ru.wikipedia.org/wiki/Цианобактерии
>Составляют значительную долю океанического фитопланктона.
Аноним 22/02/16 Пнд 13:34:46 #485 №264513 
14561372867270.jpg
14561372867291.jpg
>>264507
А когда жидкотопливные ракеты взрываются, их выключают?

Когда ракета взрывается, ее нужно не выключать, нужно от нее съебывать поскорее. Для этого существует система аварийного спасения - мачта на верхушке, которая отрывает капсулу от ракеты и уносит ее в далекие дали. А уж что там потом случится с самой ракетой - выключили ее или оставили - экипаж волновать не должно.

Шаттл на взлете взорвался только один раз причем рванул как раз бак с жидким топливом, а не бустеры. И померли все не потому, что забыли выключить двигатели, а потому что у них не было способа убежать от взрывающегося пузыря жидкого водорода и кислорода.
На SLS этот недочет исправлен, так что все у них будет хорошо, будем надеяться.
Аноним 22/02/16 Пнд 15:11:29 #486 №264520 
14561430892830.jpg
>>264513
Насадебил, это ты?
Я не против KSP. Пусть летают.
Аноним 22/02/16 Пнд 15:50:56 #487 №264527 
Почему все научные вопросы задаются в спейсаче, а на специализированной доске одни шизики?
Аноним 22/02/16 Пнд 16:02:19 #488 №264530 
>>264527
Трудно ответить однозначно, сцай более "демократичная" доска. Там принимают всех шизиков подряд, а тут сразу попросят съебать нахуй, да еще анальная мочерастия ревностно загон охраняет.
Или потому что спейсач узко тематическая доска, а в сцай все подряд сидят, поэтому бардака там больше.
Аноним 22/02/16 Пнд 16:07:29 #489 №264533 
>>264530
творческий беспорядок, мозговой штурм, острие мистики и непознаваемого
Аноним 22/02/16 Пнд 16:19:24 #490 №264534 
>>264502
1. уверен ?
2. внешние условия.
3. выполняют разную функцию.
4. система изначально не устойчива, в ней, также как и во время большого взрыва, есть дефекты, и она рзваливается в процессе эмбрионогенеза, дефференцируя ткани будущих обломков, из которых, как мозаика состоит человек. Процесс этот деликатный, и так же как на образование вселенной влияет изменения физических констант, также и с эмбрионом и дальнейшими процессами развития. Те если мамка нюхает клей, итоги удивлять не должны.
да разделом ошибся
Аноним 22/02/16 Пнд 16:20:25 #491 №264535 
>>264508
где там бустре не зажегся, и крафт повернуло и распидорасило.
Аноним 22/02/16 Пнд 16:24:21 #492 №264536 
Как используют взрывчатку в ракетных двигателях в качестве топлива?
Она же взрывается, как ни странно. Или там она постепенно горит?
Аноним 22/02/16 Пнд 16:25:21 #493 №264537 
Каким образом людям удалось поштучно подсчитать количество атомов во Вселенной?
Аноним 22/02/16 Пнд 16:53:00 #494 №264541 
>>264537
Перепись провели прост.
Никто не считал их поштучно.
Аноним 22/02/16 Пнд 17:00:40 #495 №264544 
>>264536
ДВС тоже ездит на взрывах топливной смеси.
Аноним 22/02/16 Пнд 17:39:50 #496 №264554 
>>264541
А как же 10^118
мимокрок
Аноним 22/02/16 Пнд 17:49:44 #497 №264556 
>>264554
Ну вот у тебя в деревне 10 домов, разных размеров.
В твоём доме в каждой квартире живёт в среднем по три человека.
Тебе остаётся прикинуть кол-во квартир в оставшихся домах и умножить на три - получишь примерное кол-во народу в деревне. Реальность будет отличаться в ту или иную сторону, но в целом результат будет довольно точным.
Примерно то же самое и тут.
Аноним 22/02/16 Пнд 18:19:39 #498 №264557 
>>264508
Челленджер? Нет. не слышал.
Аноним 22/02/16 Пнд 18:38:16 #499 №264560 
>>264557
А я слышал, поэтому знаю, что взорвался не сам бустер, а водородный бак.
Аноним 22/02/16 Пнд 18:41:29 #500 №264561 
>>264557
Ну с натяжкой можно считать так. А что ещё?
Аноним 22/02/16 Пнд 18:42:20 #501 №264562 
14561557401150.jpg
А почему нельзя было увести Мир на орбиту захоронения?
Аноним 22/02/16 Пнд 18:43:55 #502 №264563 
>>264562
Потому что Мир пиздец какой тяжелый, а орбита захоронения пиздец как далеко. Предположим нашелся бы безумец, который согласился бы оплатить этот банкет. И тут встает самый главный вопрос: НАХУЯ?
Аноним 22/02/16 Пнд 18:44:32 #503 №264564 
>>264560
Из-за дырявого бустера.
Аноним 22/02/16 Пнд 18:48:47 #504 №264565 
>>264563
У нас тут каждый килограмм поднятый на орбиту на вес золота, так что было бы логичным не сжигать нахуй все многолетние усилия и деньги, которые вложили в станцию, а просто заморозить её на стабильной орбите.
В будущем можно было бы экскурсии проводить, забрать оттуда какие-нибудь экспонаты или рабочие части.

Если она вся такая тяжелая, то хотя бы часть её можно было забить до отвала артефактами, отбуксировать повыше, а другую часть затопить.
Аноним 22/02/16 Пнд 18:58:54 #505 №264571 
Зачем шаттлы ходили на орбиту с экипажем из 5-8 человек? Что там вся эта компания делала?
Аноним 22/02/16 Пнд 19:03:48 #506 №264573 
>>264565
>а просто заморозить её на стабильной орбите.
Ага, главное, что бы она именно замерзла, а не разогрелась к хуям.
>В будущем можно было бы экскурсии проводить
Экскурсии на орбиту захоронения? У нас тут с суборбитальными экскурсиями беда, а уж в эти ебеня катать туристов даже не знаю, когда начнут, лет через 200.
>каждый килограмм поднятый на орбиту на вес золота
Вот об этом и речь, Мир надо поднять с 350 км на несколько тысяч, вот и считай деньги.
>Если она вся такая тяжелая
125 тонн, если что.
Короче нахуй не нужна, проще и дешевле утопить, чем заниматься подобной хуйней, с неясной перспективой.
Аноним 22/02/16 Пнд 19:05:48 #507 №264575 
>>264571
Устраивали пьянки и оргии, чем еще в космосе можно заниматься?
Тем же, чем занимаются на МКС, только время экспедиций было гораздо короче.
Эксперименты, исследования.
Аноним 22/02/16 Пнд 19:26:00 #508 №264582 
>>264573
>лет через 200.
Как раз когда на историю СССР начнут смотреть незамутненным взглядом не искаженным пропагандой. Вот тогда бы и летали к станции-реликту страны, которой уже нет.
Можно было бы продавать артефакты с Мира по цене, которая бы оплатила все расходы на поднятие с орбиты. Даже американцы могли бы купить и поднять, хотя они и так получили что хотели в виде красочного и символичного шоу в небесах.
Аноним 22/02/16 Пнд 19:45:06 #509 №264586 
>>264582
Ой, не очень понятно что будет лет через 10, а ты про 200 лет, Нострадамус хуев. Деньги-то надо было в 2001-м потратить.
Аноним 22/02/16 Пнд 20:33:24 #510 №264596 
Какой практический смысл в МКС? Ну то есть ли реальная прибыль?
Аноним 22/02/16 Пнд 20:37:24 #511 №264598 
Откуда люди знают, как выглядит наша галактика, если мы находимся внутри нее и никогда не видели ее извне?
Аноним 22/02/16 Пнд 20:43:38 #512 №264600 
В физике есть закон сохранения энергии. Нечто из ничего возникнуть не может.

Как тогда возник большой мать его взрыв из нихуя?
Аноним 22/02/16 Пнд 20:53:36 #513 №264609 
>>264598
Да, не видели. Зато нам известны угловые (относительно Земли) координаты звёзд, составляющих эту галактику, и более-менее точные расстояния до них. А дальше всё просто.
Аноним 22/02/16 Пнд 20:53:54 #514 №264611 
14561636341520.jpg
14561636341521.png
Двощь, спасай мою сраку у меня она горит! Щас смотрел РЕН ТВ и там эти пидорасы говорят что за солнцем есть планета, на луне и марсе иные цивилизации, ну тип щас нету, но были. На Церере база инопланетян. Бляди, пидорасы, суки и говно! Зачем они несут такую хуиту? Кому это нужно? И что это за пидорасы на экране у которых написано, что они профессора, физики и какие то учёные. Сука кто и для чего всё это делает, я бы этих блядей сажал бы за такой пиздёж, дольше чем барыг героиновых. Потушите мой пердак, скажите зачем они так пиздят? Я верю в вас, тут все мои друзья.
Аноним 22/02/16 Пнд 20:55:24 #515 №264612 
>>264611
Мои родители смотрят эту хуйню про рептилоидов чисто чтобы посмотреть, что эти сказочники еще придумают. Типа как фэнтези.
Аноним 22/02/16 Пнд 20:56:41 #516 №264614 
>>264596
Очередной петух решил спросить сколько бабок дают фундаментальные исследования. Иди нахуй.

>>264600
>Как тогда возник большой мать его взрыв из нихуя?
Согласно теории большого взрыва Вселенная не возникла "из нихуя", а пребывала в сингулярном состоянии, где плотность, температура и давление достигало околобесконечных значений.
>>264611
Людям нравится верить в некую хуйню, это делает их причастными к неким тайным знаниям. Ну или они слишком тупые, что бы осознать, насколько мир вокруг охуенный без всех этих сказок.

Аноним 22/02/16 Пнд 20:56:55 #517 №264615 
>>264611
Бабулькам нраица.
Аноним 22/02/16 Пнд 20:57:33 #518 №264616 
>>264596
Там в год больше тыщи исследований проводится.
Аноним 22/02/16 Пнд 21:00:22 #519 №264620 
14561640227230.jpg
>>264614
А откуда тогда взялась сингулярность? Просто я смотрю фильм дискавери про большой взрыв и там теоретические физики через слово говорят "давайте представим", "если предположить, что" и "надо просто поверить, что".

И вот смотрю я эту хуйню и честно говоря не вижу отличий от религии, раз и там и тут я должен просто что-то там себе представить, а потом поверить, что базарю посоны так все и было.
Аноним 22/02/16 Пнд 21:05:43 #520 №264623 
>>264611
Двачую этого ананаса. Тоже не понимаю зачем они все это делают. Вернее понимаю, но блядь не понимаю. Продают все. Душу, мамку, родину, ностальгию, собственый разум, чужой разум. Вообще все. И самое страшное, что это покупают и просят еще. И с этим вообще нихя не поделать. А я даже не могу стать таким же счасливым покупателем хуиты. Смотреть-радоваться и просить еще. Заебало это все.
Аноним 22/02/16 Пнд 21:09:14 #521 №264626 
14561645545200.jpg
>>264612
>>264614
>>264615
Это всё понятно ребят, я вот в полном ахуе щас продолжаю смотреть эту ересь. Но блядь у меня шаблон и пердак рвётся от того что они несут. Это не сказки, нет это даже не фантастика. Это какой то шизофринический бред, людей которые такое говорят всерьёз нужно лечить электрическим током. Ну ладно, тут разобрались не много. Теперь второй вопрос не менее важный. Почему показывают пидоров и пишут что этот физик, этот астроном, этот доктор наук и всё в таком стиле. Блядь это что серьёзно учёные мужи? Я не понимать! Сидит мудак и мне щас втираетчто он с марсианами общался на чистом английском и без акцента и их хуй от людей отличишь! Ну это же омск! Блядь я щас разъебу ящик. Анон успокой меня как то, кто эти люди?
Аноним 22/02/16 Пнд 21:11:42 #522 №264629 
>>264626
Да забей ты, блядь, это фэнтези для бабулек. Ты же не охуевашь от сеттинга какой-то вахи или старварс, где с серьезными рожами несут хуйню про жыдавев?

А ведь и ИРЛ пытались основать церковь джедаизма.
Аноним 22/02/16 Пнд 21:13:13 #523 №264631 
>>264629
>пытались
Основали.
Аноним 22/02/16 Пнд 21:13:59 #524 №264633 
>>264631
Тем более. А ты про каких-то рептилоидов.
Аноним 22/02/16 Пнд 21:14:14 #525 №264634 
14561648541990.jpg
>>264623
Анон я вот на работе сижу обедаю с мужиками со склада и они под хавчик включают эту хуебору и сидят на полном серьёзы это обсуждают. Они полностью верят всему этому, говорят что власти то скрывают, а вот РЕН ТВ нам всю правду говорят. Я как то решил у них спросить, вы серьёзно щас мужики (или тралите меня) И что ты думаешь антош, они на меня как на еблана посмотрели, тип ты чё дурак, это жи телевизор, а то физик и учёный! То есть для них всё что не говорят по ящику является истинной в последней инстанкции. А ведь таких зрителей пол страны, что с нами стало и самое главное что будет дальше?
Аноним 22/02/16 Пнд 21:17:00 #526 №264637 
>>264634
Быдло оно и в африке быдло. А на счет полстраны ты преувеличиваешь. Из молодежи НИКТО БЛЯДЬ, даже гопники не верят в рептилоидов.
Аноним 22/02/16 Пнд 21:17:44 #527 №264638 
>>264620
>там теоретические физики через слово говорят "давайте представим", "если предположить, что"
>честно говоря не вижу отличий от религии
Потому что ты тупой, очевидно. Физики мало в чем уверены на 100 процентов и допускают, что могут ошибаться, это только в религиобляди уже имеют готовые ответы на все, без всяких сомнений.

>А откуда тогда взялась сингулярность?
Ни откуда не взялась. Нет ответа на этот вопрос. Возможно Вселенная циклическая, сжимается и взрывается постоянно. Есть теории, которые вообще не предусматривают первоначального сингулярного состояния. Теория большого взрыва хоть и главенствующая, и общепринятая но не единственная.
Аноним 22/02/16 Пнд 21:18:08 #528 №264639 
>>264637
>Из молодежи НИКТО БЛЯДЬ, даже гопники не верят в рептилоидов.
Потому что всё это время гопники и были рептилоидами.
Аноним 22/02/16 Пнд 21:19:07 #529 №264640 
>>264629
Ну то художественное произведение и фан, как пастафарианство. Все, всё знают и понимают, что это развлекалово. Но тут же они на голубом глазу эту хуйню несут и обычному мимокрокодилу втирают это как документалистику с поддержкой науки и доказательной(псевдо) базой.
Аноним 22/02/16 Пнд 21:19:52 #530 №264642 
>>264638
>Физики мало в чем уверены на 100 процентов
Но при этом их теории 100%-нее религиозных, окда?

Я о том и говорю, что и те нихуя не знают и эти. Но мочёные пиздят, что их данные доказаны.

Алсо, не обзывайся, это лишь говорит о том, что ты нервничаешь, а значит не прав.
Аноним 22/02/16 Пнд 21:21:02 #531 №264643 
>>264614
>околобесконечных значений
Это глупость и словоблудие. В математике "бесконечность" - просто абстракция, число, которое больше любого другого. Гравитационная сингулярность же имеет вполне представимую физическую интерпретацию.
Аноним 22/02/16 Пнд 21:22:45 #532 №264644 
Алсо. Вот вселенная расширяется, да? Это так видно с нашей земли, закон Хаббла.

Откуда нам знать, что она действительно расширяется (на чем и строится теория взрыва), а не то, что мы попали в некую аномалию и нам за счет оптических иллюзий все эти смещения спектров мерещатся?
Аноним 22/02/16 Пнд 21:22:54 #533 №264645 
>>264642
Съеби нахуй, демагог, с тобой никто тут диспуты вести не будет, тем более, что ты рассуждаешь о научности теорий, на основе кина для тупых по дескавери.
Аноним 22/02/16 Пнд 21:23:12 #534 №264646 
>>264642
В том то и дело, что ученые оперируют доказанными данными и из-за этого не на 100 процентов уверены на счет общей картины. Доказанных данных мало.
Поп в тоже время рисутет тебе общую картину, а когда спрашиваешь его об частностях он садится в лужу и у него начинается резонерство здорового человека лол.
ПЕРЕКОТ Аноним 22/02/16 Пнд 21:24:15 #535 №264647 
Перекатывайтесь, уважаемые, потихоньку.

https://2ch.hk/spc/res/264641.html
Аноним 22/02/16 Пнд 21:24:58 #536 №264648 
>>264646
>Поп в тоже время рисутет тебе общую картину
Как и ученый. У попа вселенную создал Бог и вот она. У Ученого вселенная взорвалась и вот она.

Отличия только в частностях. За сим, я просто не вижу, где религия противоречит науке. Почему Бог не мог вызвать этот большой взрыв?
Аноним 22/02/16 Пнд 21:25:29 #537 №264649 
>>264637
Анон я по меркам двача тоже наверное быдло, мне 36 и я бородат. На бордах ещё с того. Ну почему я то не то что в это не верю, но блядь мне становится дурно когда я это вижу и слышу. Может меня закалил двощ, а говорят помойка. Не будь двоща и я бы щас Марсиан под кроватью искал бы. Двощ образовательный! Надо в школе его ввести как предмет, глядишь чё путное из детей вырастет.
Аноним 22/02/16 Пнд 21:26:19 #538 №264650 
>>264645
А ты иди нахуй.
>Здесь анонимные ученые мирового уровня критически рассмотрят любые гениальные идеи
Аноним 22/02/16 Пнд 21:34:50 #539 №264652 
>>264650
Тебе конкретно что непонятно? Почему ученые говорят так как говорят, и предлагают поверить им на слово? В 40 минутной передаче для тупых они должны с самого начала объяснить тебе всю теорию от начала до конца, с формулами, наблюдательными данными и расчетами? В кине для дебилов, серьезно? Ты конкретно что сделал, что бы прояснить для себя, почему ученые пришли к ТБВ? Или в твоем воображении они собрались, выпили и решили "да будет так"?
>>264648
>Почему Бог не мог вызвать этот большой взрыв?
Мог, только нахуя нужно некоторое божественное создание, что бы ебануть БВ, если он и без него вполне себе мог ебнуть. Бог для этого не нужен, так что нахуй бога. Выкинули за ненадобностью.
Аноним 22/02/16 Пнд 21:35:43 #540 №264654 
>>264648
>где религия противоречит науке
Везде. Ибо цели разные. Наука познает мир. Религия приручает лысых обезьян.
Аноним 22/02/16 Пнд 21:37:40 #541 №264655 
>>264648
> Почему Бог
Ну нормально же сидели, чего ты начал то?
Аноним 22/02/16 Пнд 21:40:18 #542 №264656 
>>264644
>а не то, что мы попали в некую аномалию
В какую аномалию? Что за аномалия, на каких физических законах она строится. Объясни удаление объектов во все стороны чем-то другим.
Вот наблюдения: объекты удаляются.
Самое простое и логичное объяснение наблюдениям: объекты удаляются
Что ты предлагаешь? Какое у тебя объяснение этому? Оптические иллюзии? Чем они вызваны? Какая у них природа?
Аноним 22/02/16 Пнд 21:41:23 #543 №264657 
>>264655
Религиоблядь посмотрела кино по дескавери и впервые проявило мыслительную активность. Такое бывает.
Аноним 22/02/16 Пнд 21:43:07 #544 №264661 
>>264657
>Такое бывает.
Это делает меня грустить.
Аноним 23/02/16 Втр 00:51:19 #545 №264700 
Что, если взять 6x10^24 кг куриных ножек, как будет выглядеть планета, образовавшаяся из них?
sageАноним 23/02/16 Втр 00:53:55 #546 №264701 
>>264700
Как шарик из куриных ножек
Аноним 23/02/16 Втр 01:01:43 #547 №264703 
>>264700
https://what-if.xkcd.com/4/ -- вот рассказывают, что будет, если взять моль кротов. Думаю, с ножками то же самое примерно будет.
Аноним 23/02/16 Втр 02:16:35 #548 №264748 
Играл в КСП (с рсс), там лететь до плутона 45 лет без гравиманевров. Это правда?
Аноним 23/02/16 Втр 08:15:30 #549 №264883 
>>264534
>да разделом ошибся
>>264504
Да заебали вы. Негде больше спрашивать. У меня вот тоже вопрос с генами связанный всплыл. Не в сцы же мне идти. Собственно вот. В чем такая грандиозная сложность клонирования человека по сравнению с клонированием овцы? Принцип же схожий вроде бы.
Аноним 23/02/16 Втр 08:59:30 #550 №264887 
>>264573
>Мир надо поднять с 350 км на несколько тысяч
Есть мнение, что с 350 на несколько тысяч таки проще, чем с 0 на 350. С другой стороны кучу барахла бы пришлось еще в довесок тащить с 0 на 350, это да.
Аноним 23/02/16 Втр 09:00:46 #551 №264888 
>>264598
Ну собственно о том, что земля шарообразной формы, люди узнали задолго до того, как увидели ее извне.
Аноним 23/02/16 Втр 13:31:12 #552 №264909 
>>264634
У меня батя такой. Впрочем я здесь уже батхертил по этому поводу в развернутом виде, больше не хочу. Все просто обречено, нужно это принять и жить дальше.
Аноним 23/02/16 Втр 16:21:15 #553 №264946 
>>264883
>В чем такая грандиозная сложность клонирования человека по сравнению с клонированием овцы? Принцип же схожий вроде бы.
Грандиозная сложность в том, что моралфаги-фофудьеносцы не дают. Мол, неэтично это, эксперименты над людьми и все такое.
Аноним 23/02/16 Втр 20:11:06 #554 №264986 
тред тонет, такчто фиг с ним, я хз какая там обстановка в сцы, но может там обитают компетентные, это вопрос на отдельный тред. там у них бессмертия тред есть, это явно к ним, должно быть интересно.


>>264883
это сложный вопрос, я не очень в курсе, но позволю себе пованговать.
не уверен что у них получилось с первого раза, те с одной засадки они получили 9 труппиков и одну барашку.
Второе, просто так взять ядро из клетки и запихнуть его яйцеклетку, хм интерсно было бы запихнуть в яйцеклетку барашка, получиться ли минотавр, выращенный-выношенный барашком, или негрой - хм хм не получится.
В первую очередь потому, что ядро в клетке находится в своем, так сказать клеточном режиме, и это ядро надо перевести в как бы гаметный режим.
Собственно в этом терка с стволовыми клетками и есть, они как бы еще не дифференцировались и ядро там находится в режиме близком к нужному.вроде как исследования на эмбрионах до 10 дней были разрешены, если не путаю, вроде компромисса
В третих, в экспрессии генов есть диалекты, и помимо экспрессии есть, так сказать, своя постобработка, продукта, шлифовочка, ткане и видоспецифичная. Не всегда не во всем, но есть. В чемто похоже на старые приколы с ассемблеровским кодом, когда места мало, а сказать надо много, и там начинается хитрожопия.

Поэтому один в один, как с барашкой может и не получиться. Каконить фактор роста не той системы и все приехали. Но взять соточку другую негров и ....
И понеслось, каких только замечательных экспериментов можно придумать, есть рыночек рабов, с материалом проблем нет. Зачем терять возможности, ведь кроме этого есть еще замечательные эксперименты, которые конечно же спасут человечество, там рассоспецифичные заболевания.
Зачем останавливатся, поедем к мугабе, у него там как мухи дохнут, никто не считает.

Потом в банках спермы появляются странные доноры, со странными заболеваниями, у которого теперь 500!!! питсот блять карл, детей. И вот хуй проссышь, это результат проверки подпольных тем или оплошность. Доктор съебал и возможно уже и в канаве лежит.
Четверть или треть этих детей проявляют симптомы даунизма. Не проверял эту новость(2-3 источника было, но хз кто у кого спер), продаю как есть.
Потом с родильными домами нучнут сотрудничать, ой ваш ребеночек помер, потом аборты на 8 месяце, все для науки все для войны.
И если тебе кажется что я чтото преувеличиваю, то это тебе кажется, в истории еще и не такое было.

Я тебе гарантирую, дело не в религиоблядях, и вся риторика о ебанистической пользе для человеков, миллионах спасеных жизней, белого платежеспособного населения, все это чушь собачья.
Увидишь такого псевдодухтура, пристрели его, он рак разъедающий человечество.

Этичность исследований, это не просто сказка для дураков.
А вот спасение будущих миллионов, это сказка, и поиск легких путей. И это такая дорога из красных кирпичей.
Ну и да, не переживай, стопудов этим ктото занимается.
Аноним 24/02/16 Срд 12:57:12 #555 №265127 
Можно ли вывести на орбиту человека на РН Рокот?
Аноним 24/02/16 Срд 19:25:10 #556 №265192 
>>265127
Можно, но человека жалко.
Аноним 25/02/16 Чтв 00:40:34 #557 №265286 
14563500349880.jpg
>>265127
Теоретически можно. Если верить википедии, Рокот может вытянуть на НОО от 2150 до 2300 кг, в зависимости от модификации. Американцы на заре своей космической программы летали на куда более чахлых ракетах. Скажем, Атлас - опять же, по данным той же википедии, выводил на НОО всего 1360 килограмм полезной нагрузки. И ничего, летали.
Но проблема в том, что инженерам-разработчикам капсулы Меркурий при этом приходилось экономить буквально на всем. Так, пилот сидел в теснейшей банке, упираясь ногами в одну ее стенку, а головой - в другую, и места пошевелиться у него практически не было - заденешь какую-нибудь ебучую кнопку или перекинешь нечаянно важный рубильник.
А кнопок в кабине было дохуя и больше. Дело в том, что из-за ограничений по массе инженеры не могли позволить установить на ракету почти никакой автоматики. Каждый пух и чих ракеты во время полета нужно было производить вручную, по секундомеру дергая рубильники и нажимая кнопки. Проебешь в этой фуге хоть одну ноту - в космос сегодня не полетишь. Поэтому перед полетом астронавтов дрочили до автоматизма выучивать последовательность нажатия кнопок.

А Юра наш Гагарин тем временем летал в относительно просторном шарике впоследствии в такой же шарик в модификации "Восход" умудрились затолкать аж троих человек - правда, без скафандра, кушал еду из тюбика и глазел в иллюминатор, потому что за него всю скучную механическую работу выполняла автоматика. Наши инженеры могли позволить себе такую роскошь - Восток тянул на НОО аж 4725 кг.

Так что да, как говорит >>265127-кун, летать на Рокоте в принципе можно, он даже посильнее старого Атласа будет. Да и автоматика нынче стала наверняка полегче того, чем приходилось довольствоваться тогда. Но все равно, капсула для такого полета будет тесной, одноместной и опасной. Не стоит оно того.
comments powered by Disqus

Отзывы и предложения